You are on page 1of 70

Total Marks : 200

Online Prelims TEST - 9 (SUBJECT WISE)


( InsightsIAS Mock Test Series for UPSC Preliminary Exam 2020 )

1 Consider the following statements regarding the Election Commission of India


1. The conditions of service and tenure of office of the election commissioners generally decided by
Central government
2. It conducts the elections to Parliament, Panchayats and Office of Vice President.
3. Representation of People act, 1951 fixes the term of the members of the Election commission.

Which of the statements given above is/are correct?


A. 1 only
B. 2 and 3 only
C. 1 and 3 only
D. None

Correct Answer : D

Answer Justification :

The conditions of service and tenure of office of the election commissioners and the
regional commissioners shall be determined by the president. Hence, statement 1 is
incorrect.

Article 324 of the constitution provides that the power of superintendence, direction and
control of elections to parliament, state legislatures, the office of president of India and
the office of vice-president of India shall be vested in the election commission. Thus, the
Election Commission is an all-India body in the sense that it is common to both the Central
government and the state governments. Hence, statement 2 is incorrect.

Representation of People act, 1951 doesn’t fix the term of the members of the Election
commission. Hence, statement 3 is incorrect.

2 With reference to the Union Public Service Commission (UPSC), consider the following statements:
1. The constitution of India has not specified the strength of the Commission.
2. The Constitution authorizes the parliament to determine the conditions of service of the chairman
and other members of the Commission.
3. One-half of the members of the Commission should be such persons who have held office for at least
ten years either under the Government of India or under the government of a state.

Which of the statements given above is/are correct?


A. 1 only
B. 1 and 3 only
C. 2 and 3 only
D. 1, 2 and 3

Correct Answer : B

Answer Justification :

1
Total Marks : 200
Online Prelims TEST - 9 (SUBJECT WISE)
( InsightsIAS Mock Test Series for UPSC Preliminary Exam 2020 )

The UPSC consists of a chairman and other members appointed by the president of India. The
Constitution, without specifying the strength of the Commission has left the matter to the discretion
of the president, who determines its composition. Hence, statement 1 is correct.

Further, no qualifications are prescribed for the Commission’s membership except that one-half of
the members of the Commission should be such persons who have held office for at least ten years
either under the Government of India or under the government of a state. Hence, statement 3 is
correct.

The Constitution also authorises the president to determine the conditions of service of the
chairman and other members of the Commission. Hence, statement 2 is incorrect.

3 With reference to the Finance Commission, consider the following statements:


1. It is a quasi-judicial body.
2. The Constitution authorizes the Parliament to determine the qualifications of members of the
commission and the manner in which they should be selected.
3. Fifteenth Finance Commission shall also examine whether a separate mechanism for funding of
defence and internal security ought to be set up.

Which of the statements given above is/are correct?


A. 1 and 2 only
B. 1 and 3 only
C. 2 and 3 only
D. 1, 2 and 3

Correct Answer : D

Answer Justification :

Article 280 of the Constitution of India provides for a Finance


Commission as a quasi-judicial body. It is constituted by the president
of India every fifth year or at such earlier time as he considers necessary. Hence, statement 1 is
correct.

The Finance Commission consists of a chairman and four other members to be appointed by the
president. They hold office for such period as specified by the president in his order. The
Constitution authorises the Parliament to determine the qualifications of members of the
commission and the manner in which they should be selected. Accordingly, the Parliament
has specified the qualifications of the chairman and members of the commission. Hence,
statement 2 is correct.

The Union Cabinet chaired by Prime Minister Narendra Modi has approved the proposed
amendment to enable Fifteenth Finance Commission to address serious concerns regarding the
allocation of adequate, secure and non-lapsable funds for defence and internal security of India.

2
Total Marks : 200
Online Prelims TEST - 9 (SUBJECT WISE)
( InsightsIAS Mock Test Series for UPSC Preliminary Exam 2020 )

The amendment provides that Fifteenth Finance Commission shall also examine whether a
separate mechanism for funding of defence and internal security ought to be set up and if
so how such a mechanism could be operationalized. Hence, statement 3 is correct.

https://www.pmindia.gov.in/en/news_updates/cabinet-approves-amendment-in-the-terms-of-referenc
e-for-the-fifteenth-finance-commission/

4 Consider the following statements


1. A service voter may appoint any person irrespective of his residency as his / her proxy to give vote
on his / her behalf and in his / her name at the polling station.
2. The facility of service voter is available only to the wife of a male service voter and is not available
to the husband of a female service voter.

Which of the statements given above is/are correct?


A. 1 only
B. 2 only
C. Both 1 and 2
D. Neither 1 nor 2

Correct Answer : B

Answer Justification :

Who is a ‘proxy’?

A service voter may appoint (by applying to Returning Officer in Form 13 F of Conduct of
Elections Rules, 1961 – Form available at the website of Election Commission) any person as his /
her proxy to give vote on his / her behalf and in his / her name at the polling station. The
proxy shall have to be ordinary resident of that constituency. He need not be a registered
voter but he / she must not be disqualified to be registered as a voter. Hence, statement 1 is
incorrect.

Under the existing law, this facility is available only to the wife of a male service voter and
is not available to the husband of a female service voter.

Hence, statement 2 is correct.

https://eci.gov.in/faqs/voter-electors/service-voter/faqs-service-electors-r13/

5 Consider the following statements:


1. The entire expenses including the salaries, allowances and pensions of the chairman and members
of the UPSC are subjected to vote of Parliament.
2. The conditions of service of the chairman or a member cannot be varied to his disadvantage after
his appointment.
3. The jurisdiction of UPSC can be extended by the Parliament.

3
Total Marks : 200
Online Prelims TEST - 9 (SUBJECT WISE)
( InsightsIAS Mock Test Series for UPSC Preliminary Exam 2020 )

Which of the statements given above is/are correct?


A. 1 and 2 only
B. 2 and 3 only
C. 3 only
D. 1, 2 and 3

Correct Answer : B

Answer Justification :

The entire expenses including the salaries, allowances and pensions of the chairman and members
of the UPSC are charged on the Consolidated Fund of India. Thus, they are not subject to vote of
Parliament. Hence, statement 1 is incorrect.

The conditions of service of the chairman or a member, though determined by the president, cannot
be varied to his disadvantage after his appointment. Hence, statement 2 is correct.

The additional functions relating to the services of the Union can be conferred on UPSC by the
Parliament. It can also place the personnel system of any authority, corporate body or public
institution within the jurisdiction of the UPSC. Thus, the jurisdiction of UPSC can be extended
by an act made by the Parliament. Hence, statement 3 is correct.

6 With reference to the Finance Commission, consider the following statements:


1. The recommendations made by the Finance Commission are binding on the government.
2. The commission submits its report to the union Finance minister.

Which of the statements given above is/are correct?


A. 1 only
B. 2 only
C. Both 1 and 2
D. Neither 1 nor 2

Correct Answer : D

Answer Justification :

Article 280 of the Constitution of India provides for a Finance Commission as a quasi-judicial body.
It is constituted by the president of India every fifth year or at such earlier time as he considers
necessary.

The commission submits its report to the president. He lays it before both the Houses of
Parliament along with an explanatory memorandum as to the action taken on its recommendations.
Hence, statement 2 is incorrect.

The recommendations made by the Finance Commission are only of advisory nature and hence, not
binding on the government. It is up to the Union government to implement its recommendations on

4
Total Marks : 200
Online Prelims TEST - 9 (SUBJECT WISE)
( InsightsIAS Mock Test Series for UPSC Preliminary Exam 2020 )

granting money to the states. Hence, statement 1 is incorrect.

7 With reference to the State Public Service Commission (SPSC), consider the following statements:
1. The chairman and members of the Commission hold office for a term of five years or until they
attain the age of 62 years.
2. The chairman and members of a SPSC can be removed by the governor of the concerned state.

Which of the statements given above is/are correct?


A. 1 only
B. 2 only
C. Both 1 and 2
D. Neither 1 nor 2

Correct Answer : D

Answer Justification :

The chairman and members of the Commission hold office for a term of six years or until they attain
the age of 62 years, whichever is earlier. However, they can relinquish their offices at any time by
addressing their resignation to the governor. Hence, statement 1 is incorrect.

Although the chairman and members of a SPSC are appointed by the governor, they can be
removed only by the president. The president can remove them on the same grounds and in the
same manner as he can remove a chairman or a member of the UPSC. Hence, statement 2 is
incorrect.

8 Consider the following statements regarding contesting an election in India


1. If a person is not registered as a voter in any of the constituency in India can also contest in
election.
2. If a person is registered voter in Delhi, he/she contest an election to Lok Sabha from any
constituency in the country.
3. A person confined in jail can also vote in an election.

Which of the statements given above is/are correct?


A. 1 and 2 only
B. 2 only
C. 2 and 3 only
D. None

Correct Answer : D

Answer Justification :

All the above statements are incorrect.

5
Total Marks : 200
Online Prelims TEST - 9 (SUBJECT WISE)
( InsightsIAS Mock Test Series for UPSC Preliminary Exam 2020 )

If I am not registered as a voter in any Constituency, can I contest election?

Ans. No

For contesting an election as a candidate a person must be registered as a voter. Sec 4 (d)
of Representation People Act, 1951 precludes a person from contesting unless he is an
elector in any parliamentary constituency. Section 5 (c) of R. P. Act, 1951 has a similar
provision for Assembly Constituencies. Hence, statement 1 is incorrect.

If you are a registered voter in Delhi, you can contest an election to Lok Sabha from any
constituency in the country except Assam, Lakshadweep and Sikkim, as per Section 4 (c), 4
(cc) and 4 (ccc) of the R. P. Act, 1951. Hence, statement 2 is incorrect.

Can a person confined in jail vote in an election?

Ans. No

According to section 62(5) of the Representation of the People Act, 1951, no person shall
vote at any election if he is confined in a prison, whether under a sentence of imprisonment or
transportation or otherwise, or is in the lawful custody of the police. Hence, statement 3 is
incorrect.

https://eci.gov.in/faqs/elections/contesting-for-elections/faqs-contesting-for-elections-r4/

9 Consider the following statements:


1. Joint State Public Service Commission (JSPSC) is a constitutional body.
2. Joint Public Service Commission was provided by the Government of India Act of 1919.
3. A JSPSC presents its annual performance report to each of the concerned state governors.

Which of the statements given above is/are correct?


A. 3 only
B. 2 and 3 only
C. 1 only
D. None

Correct Answer : A

Answer Justification :

The Constitution makes a provision for the establishment of a Joint State Public Service Commission
(JSPSC) for two or more states. While the UPSC and the SPSC are created directly by the
Constitution, a JSPSC can be created by an act of Parliament on the request of the state legislatures
concerned. Thus, a JSPSC is a statutory and not a constitutional body. Hence, statement 1
is incorrect.

A JSPSC presents its annual performance report to each of the concerned state governors. Each
governor places the report before the state legislature. Hence, statement 3 is correct.

6
Total Marks : 200
Online Prelims TEST - 9 (SUBJECT WISE)
( InsightsIAS Mock Test Series for UPSC Preliminary Exam 2020 )

As provided by the Government of India Act of 1919, a Central Public Service Commission was set
up in 1926 and entrusted with the task of recruiting civil servants. The Government of India Act
of 1935 provided for the establishment of not only a Federal Public Service Commission
but also a Provincial Public Service Commission and Joint Public Service Commission for
two or more provinces. Hence, statement 2 is incorrect.

10 Which of the following statements is/are the functions and powers of the National Commission for
Scheduled Castes?
1. To inquire into specific complaints with respect to the deprivation of rights and safeguards of the
SCs
2. To participate and advise on the planning process of socio-economic development of the SCs and to
evaluate the progress of their development under the Union or a state
3. The Commission has to investigate all matters relating to the constitutional and other legal
safeguards for the OBCs and the Anglo-Indian Community and report to the President upon their
working.

Select the correct answer using the code given below.


A. 1 and 2 only
B. 1 only
C. 2 and 3 only
D. 1, 2 and 3

Correct Answer : D

Answer Justification :

National Commission for Scheduled Castes (SCs) is a constitutional body in the sense that it is
directly established by Article 338 of the Constitution

The functions of the Commission are:

(a) To investigate and monitor all matters relating to the constitutional and other legal safeguards
for the SCs and to evaluate their working;

(b) To inquire into specific complaints with respect to the deprivation of rights and
safeguards of the SCs; Hence, statement 1 is correct.

(c) To participate and advise on the planning process of socio-economic development of


the SCs and to evaluate the progress of their development under the Union or a state;
Hence, statement 2 is correct.

(d) To present to the President, annually and at such other times as it may deem fit, reports upon
the working of those safeguards;

(e) To make recommendations as to the measures that should be taken by the Union or a state for
the effective implementation of those safeguards and other measures for the protection, welfare and
socio-economic development of the SCs; and

7
Total Marks : 200
Online Prelims TEST - 9 (SUBJECT WISE)
( InsightsIAS Mock Test Series for UPSC Preliminary Exam 2020 )

(f) To discharge such other functions in relation to the protection, welfare and development and
advancement of the SCs as the president may specify.

The Commission is also required to discharge similar functions with regard to the other
backward classes (OBCs) and the Anglo-Indian Community as it does with respect to the
SCs. In other words, the Commission has to investigate all matters relating to the constitutional and
other legal safeguards for the OBCs and the Anglo-Indian Community and report to the President
upon their working. Hence, statement 3 is correct.

Clause 10 of Article 338 reads as follows: “In this article, references to the Scheduled
Castes shall be construed as including references to such other backward classes as the
President may, on receipt of the report of a Commission appointed under clause (1) of
article 340 by order specify and also to the Anglo-Indian Community”

11 Which of the following is/are the functions of National Human Rights Commission (NHRC):
1. To visit jails and detention places to study the living conditions of inmates and make
recommendation thereon.
2. To review the factors including acts of terrorism that inhibit the enjoyment of human rights and
recommend remedial measures.
3. To inquire into any violation of human rights or negligence in the prevention of such violation by a
public servant, however, only on a petition presented to it or on an order of a court but not suo
motu.
4. To study treaties and other international instruments on human rights and make recommendations
for their effective implementation.

Select the correct answer using the codes given below


A. 1, 2 and 3 only
B. 1, 3 and 4 only
C. 1, 2 and 4 only
D. 1, 2, 3 and 4

Correct Answer : C

Answer Justification :

The National Human Rights Commission is a statutory but non-constitutional body. It was
established in 1993 under a legislation enacted by the Parliament, namely, the Protection of
Human Rights Act, 1993

The functions of the National Human Rights Commission (NHRC) are:

1. To inquire into any violation of human rights or negligence in the prevention of such violation by
a public servant, either suo motu or on a petition presented to it or on an order of a court.
Hence, statement 3 is incorrect.

2. To intervene in any proceeding involving allegation of violation of human rights pending before a
court.

8
Total Marks : 200
Online Prelims TEST - 9 (SUBJECT WISE)
( InsightsIAS Mock Test Series for UPSC Preliminary Exam 2020 )

3. To visit jails and detention places to study the living conditions of inmates and make
recommendation thereon. Hence, statement 1 is correct.

4. To review the constitutional and other legal safeguards for the protection of human rights and
recommend measures for their effective implementation.

5. To review the factors including acts of terrorism that inhibit the enjoyment of human
rights and recommend remedial measures. Hence, statement 2 is correct.

6. To study treaties and other international instruments on human rights and make
recommendations for their effective implementation. Hence, statement 4 is correct.

12 Which of the following Constitutional Amendment Act bifurcated the National Commission for SCs
and STs and created a separate National Commission for Scheduled Tribes?

A. 65th Constitutional Amendment Act


B. 85th Constitutional Amendment Act
C. 69th Constitutional Amendment Act
D. 89th Constitutional Amendment Act

Correct Answer : D

Answer Justification :

The National Commission for SCs and STs came into being consequent upon passing of the 65th
Constitutional Amendment Act of 1990. The Commission was established under Article 338 of the
Constitution with the objective of monitoring all the safeguards provided for the SCs and STs
under the Constitution or other laws.

In order to safeguard the interests of the STs more effectively, it was proposed to set up a separate
National Commission for STs by bifurcating the existing combined National Commission for SCs and
STs. This was done by passing the 89th Constitutional Amendment Act of 2003. This Act further
amended Article 338 and inserted a new Article 338-A in the Constitution.

Hence, option (d) is correct.

nd
13 Which of the following schedule was added by 52 Constitutional Amendment Act?

A. Seventh Schedule
B. Ninth Schedule
C. Tenth Schedule
D. Eleventh Schedule

Correct Answer : C

9
Total Marks : 200
Online Prelims TEST - 9 (SUBJECT WISE)
( InsightsIAS Mock Test Series for UPSC Preliminary Exam 2020 )

Answer Justification :

The 52nd Amendment Act of 1985 provided for the disqualification of the members of
Parliament and the state legislatures on the ground of defection from one political party to another.
For this purpose, it made changes in four Articles of the Constitution and added a new Schedule
(the Tenth Schedule) to the Constitution. This act is often referred to as the ‘anti-defection
law’. Hence, option (c) is correct.

14 With reference to the Chief Election Commissioner, consider the following statements:
1. He/she hold his/her office till the pleasure of the president.
2. The service conditions of the chief election commissioner cannot be varied to his disadvantage after
his appointment.

Which of the statements given above is/are correct?


A. 1 only
B. 2 only
C. Both 1 and 2
D. Neither 1 nor 2

Correct Answer : B

Answer Justification :

The chief election commissioner is provided with the security of tenure. He cannot be removed from
his office except in same manner and on the same grounds as a judge of the Supreme Court. Thus,
he does not hold his office till the pleasure of the president, though he is appointed by him.
Hence, statement 1 is incorrect.

The service conditions of the chief election commissioner cannot be varied to his disadvantage after
his appointment. Hence, statement 2 is correct.

15 With reference to Central Bureau of Investigation (CBI), consider the following statements:
1. The CBI is a statutory body.
2. It derives its powers from the Delhi Special Police Establishment Act, 1946.

Which of the statements given above is/are correct?


A. 1 only
B. 2 only
C. Both 1 and 2
D. Neither 1 nor 2

Correct Answer : B

Answer Justification :

10
Total Marks : 200
Online Prelims TEST - 9 (SUBJECT WISE)
( InsightsIAS Mock Test Series for UPSC Preliminary Exam 2020 )

The CBI is the main investigating agency of the Central Government.

The establishment of the CBI was recommended by the Santhanam Committee on Prevention of
Corruption (1962-1964). The CBI is not a statutory body. Hence, statement 1 is incorrect.

It derives its powers from the Delhi Special Police Establishment Act, 1946. Hence,
statement 2 is correct.

16 With reference to NITI Aayog, consider the following statements:


1. It is a non-statutory body.
2. President of India is the chairperson of NITI Aayog.
3. Vice Chairperson of NITI Aayog enjoys the rank of cabinet minister.

Which of the statements given above is/are correct?


A. 1 only
B. 2 only
C. 1 and 3 only
D. 1, 2 and 3

Correct Answer : C

Answer Justification :

The NITI Aayog, like that of the Planning Commission, was also created by an executive
resolution of the Government of India (i.e., Union Cabinet). Hence, it is also neither a
constitutional body nor a statutory body. Hence, statement 1 is correct.

The Prime Minister of India is the chairperson of NITI Aayog. Hence, statement 2 is incorrect.

Vice-Chairperson of NITI Aayog is appointed by the Prime Minister. He enjoys the rank of a
Cabinet Minister. Hence, statement 3 is correct.

17 With reference to the National Commission for Scheduled Castes (SCs), consider the following
statements:
1. It is directly established by Article 338 of the Constitution.
2. The Commission, while investigating any matter or inquiring into any complaint, has all the powers
of a criminal court trying a suit.
3. It can submit a report to President as and when it thinks necessary.

Which of the statements given above is/are correct?


A. 1 and 2 only
B. 3 only
C. 1 and 3 only
D. 1, 2 and 3

11
Total Marks : 200
Online Prelims TEST - 9 (SUBJECT WISE)
( InsightsIAS Mock Test Series for UPSC Preliminary Exam 2020 )

Correct Answer : C

Answer Justification :

The National Commission for Scheduled Castes (SCs) is a constitutional body in the sense that it is
directly established by Article 338 of the Constitution. Hence, statement 1 is correct.

The Commission, while investigating any matter or inquiring into any complaint, has all the powers
of a civil court trying a suit. Hence, statement 2 is incorrect.

The commission presents an annual report to the president. It can also submit a report as
and when it thinks necessary. Hence, statement 3 is correct. The President places all such
reports before the Parliament, along with a memorandum explaining the action taken on the
recommendations made by the Commission. The memorandum should also contain the reasons for
the
non-acceptance of any of such recommendations.

18 With reference to the Central Vigilance Commissioner, consider the following statements:
1. Central Vigilance Commissioner appointed by the President of India.
2. After his/her tenure, Central Vigilance Commissioner is not eligible for further employment under
the Central or a state government.
3. The salary, allowances and other conditions of service of the Central Vigilance Commissioner are
similar to those of the Chairman of UPSC

Which of the statements given above is/are correct?


A. 1 and 2 only
B. 2 only
C. 3 only
D. 1, 2 and 3

Correct Answer : D

Answer Justification :

The CVC is a multi-member body consisting of a Central Vigilance Commissioner (chairperson) and
not more than two vigilance commissioners.
They are appointed by the president by warrant under his hand and seal on the
recommendation of a three-member committee consisting of the prime minister as its
head, the Union minister of home affairs and the Leader of the Opposition in the Lok
Sabha. Hence, statement 1 is correct.

They hold office for a term of four years or until they attain the age of sixty five years, whichever is
earlier. After their tenure, they are not eligible for further employment under the Central
or a state government. Hence, statement 2 is correct.

The salary, allowances and other conditions of service of the Central Vigilanc
Commissioner are similar to those of the Chairman of UPSCand that of the vigilance

12
Total Marks : 200
Online Prelims TEST - 9 (SUBJECT WISE)
( InsightsIAS Mock Test Series for UPSC Preliminary Exam 2020 )

commissioner are similar to those of a member of UPSC.


But they cannot be varied to his disadvantage after his appointment.Hence, statement 3 is
correct.

19 Which of the following bodies have the leader of the opposition in Rajya Sabha, in their appointment
committees?

A. The National Human Rights Commission


B. The Central Information Commission
C. The Central Information Commission
D. CEO of NITI Aayog

Correct Answer : A

Answer Justification :

The National Human Rights Commission is a statutory (and not a constitutional) body. It was
established in 1993 under a legislation enacted by the Parliament, namely, the Protection of Human
Rights Act, 1993.

The chairman and members are appointed by the president on the recommendations of asix-
member committee consisting of the prime minister as its head, the Speaker of the Lok
Sabha, the Deputy Chairman of the Rajya Sabha, leaders of the Opposition in both the
Houses of Parliament and the Central home minister.

The CVC is a multi-member body consisting of a Central Vigilance Commissioner (chairperson) and
not more than two vigilance commissioners. They are appointed by the president by warrant
under his hand and seal on the recommendation of a three-member committee consisting
of the prime minister as its head, the Union minister of home affairs and the Leader of the
Opposition in the Lok Sabha.

The Central Information Commission was established by the Central Government in 2005. The
Commission consists of a Chief Information Commissioner and not more than ten Information
Commissioners. They are appointed by the President on the recommendation of a committee
consisting of the Prime Minister as Chairperson, the Leader of Opposition in the Lok
Sabha and a Union Cabinet Minister nominated by the Prime Minister.

CEO of NITI Aayog is appointed by Prime Minister.

Hence, option (a) is correct.

20 Expenditure from which of the following accounts/funds is/are audited by the Comptroller and
Auditor General (CAG) ?
1. Consolidated Fund of India
2. Contingency Fund of India
3. Public Account of India

13
Total Marks : 200
Online Prelims TEST - 9 (SUBJECT WISE)
( InsightsIAS Mock Test Series for UPSC Preliminary Exam 2020 )

Select the correct answer using the code given below.


A. 1 only
B. 1 and 2 only
C. 2 and 3 only
D. 1, 2 and 3

Correct Answer : D

Answer Justification :

All the statements are correct.

The duties and functions of the CAG as laid down by the Parliament and the Constitution are:
1. He audits the accounts related to all expenditure from the Consolidated Fund of India,
consolidated fund of each state and consolidated fund of each union territory having a Legislative
Assembly.
2. He audits all expenditure from the Contingency Fund of India and the Public Account of
India as well as the contingency fund of each state and the public account of each state.

21 With reference to the National Commission for Protection of Child Rights (NCPCR), consider the
following statements:
1. It is a statutory body
2. It works under the aegis of Ministry of Women and Child development
3. As per the commission, definition of child includes those up to the age of 14 years

Which of the statements given above is/are correc


A. 1 and 2 only
B. 2 only
C. 3 only
D. 1, 2 and 3

Correct Answer : A

Answer Justification :

The National Commission for Protection of Child Rights (NCPCR) is an Indian governmental
commission, established by an Act of Parliament, the Commission for Protection of Child Rights Act
in December 2005, thus is a statutory body. Hence, statement 1 is coreect.

The commission works under the aegis of Ministry of Women and Child development, GoI.
The Commission began operation a year later in March 2007. Hence, statement 2 is correct.

The Commission considers that its Mandate is "to ensure that all Laws, Policies, Programmes, and
Administrative Mechanisms are in consonance with the Child Rights perspective as enshrined in the
Constitution of India and the UN Convention on the Rights of the Child."

14
Total Marks : 200
Online Prelims TEST - 9 (SUBJECT WISE)
( InsightsIAS Mock Test Series for UPSC Preliminary Exam 2020 )

As defined by the commission,child includes those up to the age of 18 years


. Hence,
statement 3 is incorrect.

22 With reference to the Advocate General of State, consider the following statements:
1. He is the highest law officer in the state.
2. It is appointed by the governor.
3. The term of office of the advocate general is fixed for 5 years by the Constitution.

Which of the statements given above is/are correct?


A. 1 and 2 only
B. 2 only
C. 1 and 3 only
D. 1, 2 and 3

Correct Answer : A

Answer Justification :

The Constitution (Article 165) has provided for the office of the advocate general for the states. He
is the highest law officer in the state. Thus he corresponds to the Attorney General of India.
Hence, statement 1 is correct.

The advocate general is appointed by the governor. He must be a person who is qualified to be
appointed a judge of a high court. Hence, statement 2 is correct.

The term of office of the advocate general is not fixed by the Constitution . Further, the
Constitution does not contain the procedure and grounds for his removal. He holds office during the
pleasure of the governor. This means that he may be removed by the governor at any time. Hence,
statement 3 is incorrect.

23 Consider the following statements:


1. VVPATs were first used in bye-election to the Noksen Assembly Constituency of Nagaland held in
2013.
2. Conducting exit polls and publishing results of exit polls would be prohibited during the election to
Lok Sabha and State Legislative Assemblies.

Select the correct answer from the codes given below:


A. 1 only
B. 2 only
C. Both 1 and 2
D. Neither 1 nor 2

Correct Answer : C

Answer Justification :

15
Total Marks : 200
Online Prelims TEST - 9 (SUBJECT WISE)
( InsightsIAS Mock Test Series for UPSC Preliminary Exam 2020 )

VVPATs were first used in bye-election to the Noksen Assembly Constituency of Nagaland
held in 2013. Thereafter, VVPATs have been used in selected constituencies during every General
Election to State Legislative Assemblies. VVPATs were used in eight selected Parliamentary
Constituencies in the country in the 2014 Lok Sabha Election. Hence, statement 1 is correct.

According to a 2009 provision, conducting exit polls and publishing results of exit polls
would be prohibited during the election to Lok Sabha and State Legislative Assemblies.
Thus, no person shall conduct any exit poll and publish or publicise by means of the print or
electronic media or disseminate in any other manner, the result of any exit poll during the period
notified by the Election Commission in this regard. Hence, statement 2 is correct.

24 With reference to commissioners of Central Information Commission under RTI Act:


1. They should not be a Member of Parliament or Member of the Legislature of any State or Union
Territory.
2. They are appointed by the President on the recommendation of a committee consisting of the Prime
Minister as Chairperson, the Leader of Opposition in the Lok Sabha and a Union Cabinet Minister
nominated by the Prime Minister.
3. Commissioners are appointed for duration of five years until they attain the age of 62 years,
whichever is earlier.

Which of the statements given above is/are correct?


A. 1 and 2 only
B. 2 only
C. 3 only
D. 1, 2 and 3

Correct Answer : A

Answer Justification :

The Commission consists of a Chief Information Commissioner and not more than ten Information
Commissioners. They are appointed by the President on the recommendation of a
committee consisting of the Prime Minister as Chairperson, the Leader of Opposition in
the Lok Sabha and a Union Cabinet Minister nominated by the Prime Minister. Hence,
statement 2 is correct.

They should not be a Member of Parliament or Member of the Legislature of any State or
Union Territory. They should not hold any other office of profit or connected with any political
party or carrying on any business or pursuing any profession. Hence, statement 1 is correct.

The Chief Information Commissioner and an Information Commissioner hold office for a
term of 5 years or until they attain the age of 65 years, whichever is earlier. They are not
eligible for reappointment. Hence, statement 3 is incorrect.

25 With reference to Attorney General of India, consider the following statements:


1. He receives such remuneration as the president may determine.

16
Total Marks : 200
Online Prelims TEST - 9 (SUBJECT WISE)
( InsightsIAS Mock Test Series for UPSC Preliminary Exam 2020 )

2. He must be a person who is qualified to be appointed a judge of the Supreme Court.

Which of the statements given above is/are correct?


A. 1 only
B. 2 only
C. Both 1 and 2
D. Neither 1 nor 2

Correct Answer : C

Answer Justification :

The Constitution (Article 76) has provided for the office of the Attorney General for India1. He is the
highest law officer in the country.

The Attorney General (AG) is appointed by the president. He must be a person who is
qualified to be appointed a judge of the Supreme Court. In other words, he must be a citizen
of India and he must have been a judge of some high court for five years or an advocate of some
high court for ten years or an eminent jurist, in the opinion of the president. Hence, statement 1
is correct.

The remuneration of the AG is not fixed by the Constitution. He receives such remuneration as the
president may determine. Hence, statement 2 is correct.

26 With reference to the Central Vigilance Commission, which of the following statements is/are
correct?
1. It was established in 1964 by an executive resolution of the Central government
2. Presently, it is a statutory body.

Select the correct answer using the codes given below:


A. 1 only
B. 2 only
C. Both 1 and 2
D. Neither 1 nor 2

Correct Answer : C

Answer Justification :

The Central Vigilance Commission (CVC) is the main agency for preventing corruption in the
Central government. It was established in 1964 by an executive resolution of the Central
government. Its establishment was recommended by the Santhanam Committee on Prevention of
Corruption (1962–64). Hence, statement 1 is correct.

Thus, originally the CVC was neither a constitutional body nor a statutory body. Later, in
2003, the Parliament enacted a law conferring statutory status on the CVC. Hence,

17
Total Marks : 200
Online Prelims TEST - 9 (SUBJECT WISE)
( InsightsIAS Mock Test Series for UPSC Preliminary Exam 2020 )

statement 2 is correct.

27 With reference to the State Information Commission, consider the following statements:
1. The Commission consists of a State Chief Information Commissioner and not more than five State
Information Commissioners.
2. Members of the commission are appointed by the President of India.
3. They are not eligible for reappointment.

Which of the statements given above is/are correct?


A. 1 only
B. 1 and 2 only
C. 3 only
D. 2 and 3 only

Correct Answer : C

Answer Justification :

The Right to Information Act of 2005 provides for the creation of not only the Central Information
Commission but also a State Information Commission at the state level.

The Commission consists of a State Chief Information Commissioner and not more than ten State
Information Commissioners. Hence, statement 1 is incorrect.

They are appointed by the Governor on the recommendation of a committee consisting of the
Chief Minister as Chairperson, the Leader of Opposition in the Legislative Assembly and a State
Cabinet Minister nominated by the Chief Minister. Hence, statement 2 is incorrect.

The State Chief Information Commissioner and a State Information Commissioner hold office for a
term of 5 years or until they attain the age of 65 years, whichever is earlier. They are not eligible
for reappointment. Hence, statement 3 is correct.

28 With reference to the Indian Council of Agricultural Research (ICAR), consider the following
statements:
1. It is an autonomous organization under the Ministry of Agriculture and Farmers Welfare.
2. Prime minister serves as its president.
3. Recently, it has published an Integrated Mobile App called KISAAN (Krishi Integrated Solution for
Agri Apps Navigation) for Farmers of country.

Which of the statement(s) given above is/are correct?


A. 2 and 3 only
B. 1 and 3 only
C. 1 and 2 only
D. 1, 2 and 3

18
Total Marks : 200
Online Prelims TEST - 9 (SUBJECT WISE)
( InsightsIAS Mock Test Series for UPSC Preliminary Exam 2020 )

Correct Answer : B

Answer Justification :

The Indian Council of Agricultural Research (ICAR) is an autonomous body responsible for
coordinating agricultural education and research in India. Itreports to the Department of
Agricultural Research and Education, Ministry of Agriculture . Hence, statement 1 is
correct.

The Union Minister of Agriculture serves as its president. Hence, statement 2 is incorrect.

In 2019 ICAR has published an Integrated Mobile App called KISAAN (Krishi Integrated
Solution for Agri Apps Navigation) for Farmers of country.Mobile App has interface in 12
Indian Languages. Hence, statement 3 is correct.

29 Consider the following statements:


1. Indian Space Research Organization (ISRO) set up the Thumba Equatorial Rocket Launching
Station (TERLS) in Thiruvananthapuram for upper atmospheric research.
2. APSARA is the first nuclear reactor in Asia to have achieved criticality.

Which of the statements given above is/are correct?


A. 1 only
B. 2 only
C. Both 1 and 2
D. Neither 1 nor 2

Correct Answer : B

Answer Justification :

India decided to go to space when Indian National Committee for Space Research (INCOSPAR) was
set up by the Government of India in 1962. With the visionary Dr Vikram Sarabhai at its helm,
INCOSPAR set up the Thumba Equatorial Rocket Launching Station (TERLS) in
Thiruvananthapuram for upper atmospheric research. Indian Space Research Organisation,
formed in 1969, superseded the erstwhile INCOSPAR. Hence, statement 1 is incorrect.

APSARA is the first nuclear reactor in Asia to have achieved criticality on August 4, 1956. It
is a pool type reactor of 1 MW power with a highly enriched uranium as fuel (4.5 kg) in the form of
plates. Light water was used as both moderator and coolant. Hence, statement 2 is correct.

http://www.barc.gov.in/about/index.html#

https://www.isro.gov.in/about-isro

30 Consider the following statements:


1. Telecom Regulatory Authority of India is a non-statutory executive body established by an executive

19
Total Marks : 200
Online Prelims TEST - 9 (SUBJECT WISE)
( InsightsIAS Mock Test Series for UPSC Preliminary Exam 2020 )

resolution.
2. The National Commission for Safai Karamcharis (NCSK) was constituted as a statutory body, but
presently it is working as a non-statutory body.
3. The National Commission for Denotified, Nomadic and Semi-Nomadic Tribes is a national
commission set under the Ministry of Tribal Affairs.

Which of the statements given above is/are correct?


A. 2 only
B. 1 and 3
C. 3 only
D. 1, 2 and 3

Correct Answer : A

Answer Justification :

The Telecom Regulatory Authority of India (TRAI) was, thus, established with effect from
20th February 1997 by an Act of Parliament, called the Telecom Regulatory Authority of India
Act, 1997, to regulate telecom services, including fixation/revision of tariffs for telecom services
which were earlier vested in the Central Government. Hence, statement 1 is incorrect.

The National Commission for Safai Karamcharis (NCSK) was constituted on 12th August,
1994 as a statutory body by an Act of Parliamentv iz. ‘National Commission for Safai
Karamcharis Act, 1993’, for a period of three years i.e. up to 31st March, 1997.

With the lapsing of the “The National Commission for Safai Karamcharis Act, 1993” w.e.f.
29.2.2004, the Commission is acting as a Non-Statutory body of the Ministry of Social Justice
and Empowerment whose tenure is extended from time to time through Government Resolutions.

Hence, statement 2 is correct.

The National Commission for Denotified, Nomadic and Semi-Nomadic Tribes is a national
commission set under the Ministry of Social Justice and Empowermentto study various
Hence,
developmental aspects of denotified and nomadic or semi-nomadic tribes in India.
statement 3 is incorrect.

https://main.trai.gov.in/about-us/history

https://ncsk.nic.in/about-us/about-ncsk

31 With reference to National Commission for Women, consider the following statements:
1. It is a statutory body.
2. It take suo-moto notice of matters relating to deprivation of women's rights.
3. Among the five members of the commission at least one Member each shall be from amongst
persons belonging to the Scheduled Castes and Scheduled Tribes respectively.

Which of the statements given above is/are correct?

20
Total Marks : 200
Online Prelims TEST - 9 (SUBJECT WISE)
( InsightsIAS Mock Test Series for UPSC Preliminary Exam 2020 )

A. 1 only
B. 2 and 3
C. 1 and 3
D. 1, 2 and 3

Correct Answer : D

Answer Justification :

The National Commission for Women was set up as statutory body in January 1992 under
the National Commission for Women Act, 1990 ( Act No. 20 of 1990 of Govt.of India ) to :

review the Constitutional and Legal safeguards for women ;

recommend remedial legislative measures ;

facilitate redressal of grievances and

advise the Government on all policy matters affecting women.

Hence, statement 1 is correct.

The Commission shall look into complaints and take suo moto notice of matters relating to:-

1. deprivation of women's rights. Hence, statement 2 is correct.

2. non-implementation of laws enacted to provide protection to women and also to achieve the
objective of equality and development,

3. non-compliance of policy decisions,guidelines or instructions aimed at mitigating hardships


and ensuring welfare and providing relief to women, and take up the issues arising out of
such matters with appropriate authorities.

http://ncw.nic.in/commission/about-us

http://ncw.nic.in/commission/about-us/constitution

http://ncw.nic.in/commission/about-us/mandate

The Commission shall consist of :-

21
Total Marks : 200
Online Prelims TEST - 9 (SUBJECT WISE)
( InsightsIAS Mock Test Series for UPSC Preliminary Exam 2020 )

1. A Chairperson, committed to the cause of women, to be nominated by the Central


Government.

2. five Members to be nominated by the Central Government from amongst persons of ability,
integrity and standing who have had experience in law or legislation, trade unionism,
management of an industry potential of women, women's voluntary organisations ( including
women activist ), administration, economic development, health, education or social welfare;
Provided that at least one Member each shall be from amongst persons belonging to
the Scheduled Castes and Scheduled Tribes respectively. Hence, statement 3 is
correct.

32 Consider the following statements:


1. India Meteorological Department (IMD) works under the Ministry of Environment, Forest and
Climate Change (MoEFCC)
2. It is the duty of the Competition Commission of India (CCI) to eliminate practices having adverse
effect on competition, promote and sustain competition, protect the interests of consumers and
ensure freedom of trade in the markets of India

Which of the statements given above is/are correct?


A. 1 only
B. 2 only
C. Both 1 and 2
D. Neither 1 nor 2

Correct Answer : B

Answer Justification :

The India Meteorological Department (IMD) is an agency of the Ministry of Earth


Sciences of the Government of India. It is the principal agency responsible for
meteorological observations, weather forecasting and seismology. Hence, statement 1 is
incorrect.

IMD is also one of the six Regional Specialised Meteorological Centres of the World Meteorological
Organization. It has the responsibility for forecasting, naming and distribution of warnings
for tropical cyclones in the Northern Indian Ocean region, including the Malacca Straits, the Bay of
Bengal, the Arabian Sea and the Persian Gulf.

Competition Commission of India is a statutory body of the Government of India responsible for
enforcing The Competition Act, 2002 throughout India and to prevent activities that have an
appreciable adverse effect on competition in India.

It is the duty of the Commission to eliminate practices having adverse effect on competition,
promote and sustain competition, protect the interests of consumers and ensure freedom
of trade in the markets of India. The Commission is also required to give an opinion on
competition issues on a reference received from a statutory authority established under any law and

22
Total Marks : 200
Online Prelims TEST - 9 (SUBJECT WISE)
( InsightsIAS Mock Test Series for UPSC Preliminary Exam 2020 )

to undertake competition advocacy, create public awareness and impart training on competition
issues. Hence, statement 2 is correct.

http://imd.gov.in/pages/about_organisation.php

https://www.cci.gov.in/about-cci

33 Consider the following statements:


1. The Central Electricity Authority of India (CEA) is a statutory organization .
2. Council of Scientific and Industrial Research operates as an autonomous body through the Societies
Registration Act, 1860

Which of the statements given above is/are correct?


A. 1 only
B. 2 only
C. Both 1 and 2
D. Neither 1 nor 2

Correct Answer : C

Answer Justification :

Central Electricity Authority (CEA) is an organization originally constituted under Section 3(1)
of the repealed Electricity (Supply) Act, 1948, since substituted by Section 70 of the
Electricity Act, 2003. It was established as a part-time body in 1951 and made a full-time body in
1975. The functions and duties of CEA are delineated under Section 73 of the Electricity Act, 2003.
Hence, statement 1 is correct.

The Council of Scientific and Industrial Research (CSIR) was established by the Government
of India in September of 1942 as an autonomous body that has emerged as the largest research
and development organisation in India. Although it is mainly funded by the Ministry of Science and
Technology, it operates as an autonomous body through the Societies Registration Act, 1860.
Hence, statement 2 is correct.

http://cea.nic.in/aboutus.html

34 Consider the following statements regarding National Foundation for Communal Harmony
1. It is an autonomous body under the administrative control of the Union Home Ministry.
2. It provides financial assistance to the child victims of societal violence for their care, education and
training.

Which of the statements given above is/are correct?


A. 1 only
B. 2 only
C. Both 1 and 2
D. Neither 1 nor 2

23
Total Marks : 200
Online Prelims TEST - 9 (SUBJECT WISE)
( InsightsIAS Mock Test Series for UPSC Preliminary Exam 2020 )

Correct Answer : C

Answer Justification :

The National Foundation for Communal Harmony (NFCH) was set up in 1992. It is an
autonomous body under the administrative control of the Union Home Ministry. It
promotes communal harmony, fraternity and national integration. Hence Statement 1 is
correct.

The activities undertaken by the NFCH are mentioned below:

1. To provide financial assistance to the child victims of societal violence for their care,
education and training, aimed at their effective rehabilitation Hence Statement 2 is
correct.

2. To promote communal harmony and national integration by organising variety of activities either
independently or in association with educational institutions, NGOs & other organisations

3. To conduct studies and grant scholarships to institutions / scholars for conducting studies

35 Which of the following is/are the principles of Indian Foreign Policy?


1. Promotion of World Peace
2. Anti-Colonialism
3. Non-Alignment
4. Links with Commonwealth

Select the correct answer using the code given below


A. 1 and 2 only
B. 1, 2 and 3 only
C. 2, 3 and 4 only
D. 1, 2, 3 and 4

Correct Answer : D

Answer Justification :

Principles of Indian Foreign Policy

• Promotion of World Peace

• Anti-Colonialism

• Anti-Racialism

• Non-Alignment

• Panchsheel

24
Total Marks : 200
Online Prelims TEST - 9 (SUBJECT WISE)
( InsightsIAS Mock Test Series for UPSC Preliminary Exam 2020 )

• Afro-Asian Bias

• Links with Commonwealth

• Support to the UNO

Hence, option (d) is correct.

36 Consider the following statements


1. India’s Look East Policy was first initiated by I.K. Gujral
2. No South Asian country should allow its territory to be used against the interest of another country
of the region is one of the features of Gujral Doctrine

Which of the statements given above is/are correct?


A. 1 only
B. 2 only
C. Both 1 and 2
D. Neither 1 nor 2

Correct Answer : B

Answer Justification :

India’s ‘Look East’ Policy was first initiated in 1992 by the then Prime Minister P.V.
Narasimha Rao. Hence Statement 1 is incorrect.

Since then, the policy has been one of the cornerstones of India’s foreign policy. The policy is a
strategic shift in India’s vision of the world and India’s place in the evolving global economy. It lays
emphasis on improving cooperation with India’s neighbouring south-east and East Asian countries.

Gujral doctrine is a five-point roadmap to guide the conduct of India’s foreign relations with its
immediate neighbours. These five principles are as follows:

1. With the neighbors like Bangladesh, Bhutan, Maldives, Nepal and Sri Lanka, India should not ask
for reciprocity, but give to them what it can in good faith.

2. No South Asian country should allow its territory to be used against the interest of
another country of the region. Hence Statement 2 is correct.

3. No country should interfere in the internal affairs of another country.

4. All South Asian countries should respect each other’s territorial integrity and sovereignty.

5. All South Asian countries should settle all their disputes through peaceful bilateral negotiations.

25
Total Marks : 200
Online Prelims TEST - 9 (SUBJECT WISE)
( InsightsIAS Mock Test Series for UPSC Preliminary Exam 2020 )

37 Consider the following statements regarding Nuclear Doctrine of India


1. Nuclear retaliatory attacks can only be authorised by the civilian political leadership
2. Non-use of nuclear weapons against non-nuclear weapon states
3. Building and maintaining a credible minimum deterrent.

Which of the statements given above is/are correct?


A. 1 and 2 only
B. 2 and 3 only
C. 3 only
D. 1, 2 and 3

Correct Answer : D

Answer Justification :

All the statements given above are correct.

Nuclear Doctrine of India

India adopted its nuclear doctrine in 2003. The salient features of this doctrine are as follows:

1. Building and maintaining a credible minimum deterrent.

2. A posture of “No First Use” – nuclear weapons will only be used in retaliation against a nuclear
attack on Indian territory or on Indian forces anywhere.

3. Nuclear retaliation to a first strike will be massive and designed to inflict unacceptable damage.

4. Nuclear retaliatory attacks can only be authorised by the civilian political leadership through the
Nuclear Command Authority.

5. Non-use of nuclear weapons against non-nuclear weapon states.

6. However, in the event of a major attack against India, or Indian forces anywhere, by biological or
chemical weapons, India will retain the option of retaliating with nuclear weapons.

7. A continuance of strict controls on export of nuclear and missile related materials and
technologies, participation in the Fissile Material Cutoff Treaty negotiations, and continued
observance of the moratorium on nuclear tests.

8. Continued commitment to the goal of a nuclear-weapon-free world, through global, verifiable and
non-discriminatory nuclear disarmament.

38 Cabinet committee on Security consists of which of the following members?


1. Prime Minister
2. Minister of Home Affairs
3. Minister of External Affairs
4. Ministry of Road Transport and Highways

26
Total Marks : 200
Online Prelims TEST - 9 (SUBJECT WISE)
( InsightsIAS Mock Test Series for UPSC Preliminary Exam 2020 )

Select the correct answer using the code given below


A. 1 and 3 only
B. 1, 2 and 3 only
C. 1 and 2 only
D. 1, 2, 3 and 4

Correct Answer : B

Answer Justification :

The Cabinet Committee on Security (CCS) of the Central Government of India discusses, debates
and is the final decision-making body on senior appointments in the national security apparatus
,defence policy and expenditure, and generally all matters of India's national security. The CCS is
chaired by the Prime Minister of India. The CCS consists of the following members:

Prime Minister

Minister of Home Affairs

Minister of Defence

Minister of External Affairs

Minister of Finance and Corporate Affairs.

Hence, option (b) is correct.

39 Which of these laws in India have a bearing on health?


1. Narcotic Drugs and Psychotropic Substances Act, 1985
2. The Prevention of Food Adulteration Act, 1954
3. Bio-Medical Waste (Management and Handling) Rules, 1998
4. Consumer Protection Act, 1986

Select the correct answer using the codes below.


A. 1 and 2 only
B. 3 and 4 only
C. 1, 2 and 3 only
D. 1, 2, 3 and 4

Correct Answer : D

Answer Justification :

All the above laws have bearing on Health

The Act prohibits a person to produce/manufacture/cultivate, possess, sell, purchase, transport,

27
Total Marks : 200
Online Prelims TEST - 9 (SUBJECT WISE)
( InsightsIAS Mock Test Series for UPSC Preliminary Exam 2020 )

store, and/or consume any narcotic drug or psychotropic substance which has a direct bearing on
personal and social health.

Food adulteration, such as mixing of ammonia in milk, significantly risks public health and is thus
regulated by the Act.

Bio-medical wastes emitted by hospitals and other such institutions are regulated by this act, which
could become a local menace by leaching into groundwater, spread microorganisms etc.

Suppose a wrong medicine was prescribed by a hospital to patient, consuming which causes
immense damage to the patient. The patient, as a consumer, can raise this issue in the consumer
court and get adequate compensation.

40 How is the World Bank Group (WBG) President selected?

A. Nominated by the largest shareholder of WBG subject to confirmation by the Board of


Governors
B. Elected on a majority basis by the shareholder member states of the WBG
C. Elected by the United Nations General Assembly (UNGA) on a two-thirds majority basis
D. Appointed by the UN Secretary General subject to approval by the Board of Governors

Correct Answer : A

Answer Justification :

Traditionally, the Bank President has always been a U.S. citizen nominated by the
President of the United States, the largest shareholder in the bank. The nominee is subject
to confirmation by the Board of Governors, to serve for a five-year, renewable term. The president is
responsible for chairing the meetings of the Boards of Directors and for overall management of the
World Bank Group. Hence, option (a) is correct.

41 Consider the following statements regarding Financial Action Task Force (FATF)
1. It was set up in 1989 by International Monetary Fund with headquarters in Paris.
2. FATF acts as an international watchdog on issues of money-laundering and financing of terrorism.
3. The Gulf Cooperation Council (GCC) and the European Commission (EC) are members of it.

Which of the statements given above is/are correct?


A. 1 and 3 only
B. 2 and 3 only
C. 1 and 2 only
D. 1, 2 and 3

Correct Answer : B

Answer Justification :

28
Total Marks : 200
Online Prelims TEST - 9 (SUBJECT WISE)
( InsightsIAS Mock Test Series for UPSC Preliminary Exam 2020 )

Financial Action Task Force was set up in 1989 by the G7 countries with headquarters in
Paris. Hence Statement 1 is incorrect.

FATF acts as an “international watchdog‟ on issues of money-laundering and financing of


terrorism. It has 37 members, which include all 5 permanent members of the Security Council and
other countries with economic influence. Hence Statement 2 is correct.

Two regional organisations, the Gulf Cooperation Council (GCC) and the European
Commission (EC) are also its members. Hence Statement 3 is correct.

https://www.fatf-gafi.org/about/

42 Consider the following statements regarding BRICS


1. South Africa joined BRICS in 2010
2. Last summit of BRICS was held at China.
3. BRICS established The Asian Infrastructure Investment Bank

Which of the statements given above is/are correct?


A. 1 only
B. 2 only
C. 1 and 3 only
D. 1, 2 and 3

Correct Answer : A

Answer Justification :

BRICS is the acronym coined for an association of five major emerging national economies: Brazil,
Russia, India, China and South Africa. Originally the first four were grouped as "BRIC" (or "the
BRICs"), before the induction of South Africa in 2010. Hence Statement 1 is correct.

Last summit of BRICS was held at South Africa. Hence Statement 2 is incorrect.

The proposal for the creation of an "Asian Infrastructure Investment Bank" was first made by
the Vice Chairman of the China Center for International Economic Exchanges, a Chinese thinktank,
at the Bo'ao Forum in April 2009. The initiative was officially launched by Chinese President
Xi Jinping on a state visit to Indonesia in October 2013. Hence Statement 3 is incorrect.

43 Consider the following statements


1. The Nuclear Suppliers Group (NSG) was set up in 1974 as a reaction to India’s nuclear tests to stop
what it called the misuse of nuclear material meant for peaceful purposes.
2. The NSG works under the principle of unanimity and even one country’s vote against prospective
member will scuttle its bid.
3. India, Pakistan, Israel, South Sudan and North Korea are among the four UN member states which
have not signed the NPT.

29
Total Marks : 200
Online Prelims TEST - 9 (SUBJECT WISE)
( InsightsIAS Mock Test Series for UPSC Preliminary Exam 2020 )

Which of the following statements given above is/are correct?


A. 1 only
B. 2 and 3 only
C. 1 and 2 only
D. 1, 2 and 3

Correct Answer : C

Answer Justification :

The NSG was founded in response to the Indian nuclear test in May 1974 and first met in
November 1975. The test demonstrated that certain non-weapons specific nuclear technology could
be readily turned to weapons development. Hence Statement 1 is correct.

The NSG looks after critical issues relating to nuclear sector and its members are allowed to trade
in and export nuclear technology. The group works under the principle of unanimity and even
one country's vote against India will scuttle its bid. Hence Statement 2 is correct.

As of August 2016, 191 states have adhered to the treaty, though North Korea, which
acceded in 1985 but never came into compliance, announced its withdrawal from the NPT in 2003,
following detonation of nuclear devices in violation of core obligations. Hence Statement 3 is
incorrect.

https://en.wikipedia.org/wiki/Nuclear_Suppliers_Group

44 Consider the following statements regarding Group of 7 (G7) countries


1. G7 is a group consisting of Canada, France, Germany, Italy, Japan, the United Kingdom, and the
United States.
2. Canada has least population amongst G7 countries
3. G7 countries represent more than 40% of the net global wealth.

Which of the statements given above is/are correct?


A. 1 only
B. 1 and 3 only
C. 2 and 3 only
D. 1, 2 and 3

Correct Answer : D

Answer Justification :

All the statements given above are correct.

The Group of 7 (G7) is a group consisting of Canada, France, Germany, Italy, Japan, the United
Kingdom, and the United States. The European Union is also represented within the G7.

30
Total Marks : 200
Online Prelims TEST - 9 (SUBJECT WISE)
( InsightsIAS Mock Test Series for UPSC Preliminary Exam 2020 )

As of 2018, the seven countries involved represent 58% of the global net wealth ($317
trillion) and more than 46% of the global gross domestic product (GDP) based on nominal values,
and more than 32% of the global GDP based on purchasing power parity.

Canada has least population amongst G7 countries (35,467,000).

https://en.wikipedia.org/wiki/Group_of_Seven

45 The Global Infrastructure Facility, sometimes seen in the news, is an initiative by

A. The International Monetary Fund


B. The World Bank
C. The World Economic Forum
D. The United Nations Conference on Trade and Development

Correct Answer : B

Answer Justification :

The Global Infrastructure Facility (GIF) is a partnership among governments, multilateral


development banks, private sector investors, and financiers. It is designed to provide a new
way to collaborate on preparing, structuring, and implementing complex projects that no single
institution could handle on its own.

The World Bank (WB) has launched the Global Infrastructure Facility.

https://www.globalinfrafacility.org/what-is-the-gif

46 The world's first IAEA Low Enriched Uranium (LEU) bank had established in

A. Japan
B. United States
C. France
D. Kazakhstan

Correct Answer : D

Answer Justification :

The IAEA LEU Bank will be a reserve of LEU owned and controlled by the IAEA, and a
mechanism of last resort for Member States in case the supply of LEU to a nuclear power plant is
disrupted due to exceptional circumstances and the Member State is unable to secure LEU from the
commercial market or by any other means.

World’s first IAEA was established in Kazakhstan.

31
Total Marks : 200
Online Prelims TEST - 9 (SUBJECT WISE)
( InsightsIAS Mock Test Series for UPSC Preliminary Exam 2020 )

https://www.iaea.org/topics/iaea-low-enriched-uranium-bank

47 Consider the following statements


1. Muslims, Sikhs, Christians, Buddhists, Jain and Zorastrians (Parsis) have been notified as minority
communities under of the National Commission for Minorities Act, 1992.
2. Prime Minister's Twenty Point Programme for minorities is a programme launched by Indian
government in 2006 for welfare of religious minorities.
3. National Minorities Development & Finance Corporation (NMDFC) aims to promote the economic &
developmental activities for the benefit of all sections of notified Minorities

Which of the statements given above is/are correct?


A. 2 only
B. 1 only
C. 1 and 3 only
D. 1, 2 and 3

Correct Answer : B

Answer Justification :

Muslims, Sikhs, Christians, Buddhists, Jain and Zorastrians (Parsis) have been notified as minority
communities under of the National Commission for Minorities Act, 1992. Hence Statement 1 is
correct.

Prime Minister's 15 point programme for minorities is a programme launched by Indian


government in 2006 for welfare of religious minorities. Hence Statement 2 is incorrect.

National Minorities Development & Finance Corporation (NMDFC) aims to promote the economic &
developmental activities for the benefit of backward sections of notified minorities. Hence
Statement 3 is incorrect.

48 Consider the following statements


1. Organisation of Islamic Cooperation (OIC) is a regional organization within European Union.
2. Arab League is the collective voice of the Muslim world and works to safeguard and protect the
interests of the Muslim world in the spirit of promoting international peace and harmony

Which of the statements given above is/are correct?


A. 1 only
B. 2 only
C. Both 1 and 2
D. Neither 1 nor 2

Correct Answer : D

Answer Justification :

32
Total Marks : 200
Online Prelims TEST - 9 (SUBJECT WISE)
( InsightsIAS Mock Test Series for UPSC Preliminary Exam 2020 )

Organisation of Islamic Cooperation (OIC) is a regional organization but not within European
Union. Hence Statement 1 is incorrect.

Organisation of Islamic Cooperation (OIC) is the collective voice of the Muslim world and works to
safeguard and protect the interests of the Muslim world in the spirit of promoting international
peace and harmony. Hence Statement 2 is incorrect.

https://www.oic-oci.org/page/?p_id=52&p_ref=26&lan=en

https://en.wikipedia.org/wiki/Organisation_of_Islamic_Cooperation

49 Consider the following statements


1. The Sangeet Natak Akademi is one of the first National Academy of the arts set-up by the Republic
of India.
2. The Sangeet Natak Akademi is presently an autonomous body of the Ministry of Culture,
Government of India and is fully funded by the Government for implementation of its schemes and
programmes.

Which of the statement given above is/are correct?


A. 1 only
B. 2 only
C. Both 1 and 2
D. Neither 1 nor 2

Correct Answer : C

Answer Justification :

Both the statements are correct.

The Sangeet Natak Akademi - India's national academy for music, dance and drama - is the first
National Academy of the arts set-up by the Republic of India. It was created by a resolution of
the (then) Ministry of Education, Government of India, dated 31 May 1952 notified in the Gazette of
India of June 1952.

The Akademi became functional the following year, with the appointment of its first Chairman, Dr
P.V. Rajamannar, and the formation of its all-India council of representatives, the General Council.

The Sangeet Natak Akademi is presently an Autonomous Body of the Ministry of Culture,
Government of India and is fully funded by the Government for implementation of its
schemes and programmes.

https://www.sangeetnatak.gov.in/sna/introduction.php

50 Section 8 of the Representation of the People (RP) Act, 1951 that is frequently in news concerns

33
Total Marks : 200
Online Prelims TEST - 9 (SUBJECT WISE)
( InsightsIAS Mock Test Series for UPSC Preliminary Exam 2020 )

A. Paid advertisements for election campaign.


B. Division of seats in the Upper house.
C. Horse trading in council elections.
D. Disqualification on conviction for long sentences.

Correct Answer : D

Answer Justification :

Section 8 disqualifies a person convicted with a sentence of two years or more from
contesting elections. But those under trial continued to be eligible to contest elections. The Lily
Thomas case (2013), however, ended this unfair advantage.

Currently, under the Representation of Peoples (RP) Act, lawmakers cannot contest elections only
after their conviction in a criminal case.

51 Consider the following statements


1. Swadhar is a central sector scheme for integrated services to women in difficult circumstances like
destitute widows, women prisoners released from jail and without family support, women survivors
of natural disasters etc.
2. Sakhi is a one-stop centre for providing medical, legal and rehabilitation facilities for women
subjected to any kind of violence.

Which of the statements given above is/are not correct?


A. 1 only
B. 2 only
C. Both 1 and 2
D. Neither 1 nor 2

Correct Answer : D

Answer Justification :

Both the statements are correct.

The scheme envisions a supportive institutional framework for women victims of difficult
circumstances so that they could lead their life with dignity and conviction. It envisages that
shelter, food, clothing, and health as well as economic and social security are assured for such
women. It also envisions that the special needs of these women are properly taken care of and
under no circumstances they should be left unattended or abandoned which could lead to their
exploitation and desolation

http://vikaspedia.in/social-welfare/women-and-child-development/women-development-1/swadhar

http://vikaspedia.in/social-welfare/women-and-child-development/women-development-1/poorna-sha
kti-kendras

34
Total Marks : 200
Online Prelims TEST - 9 (SUBJECT WISE)
( InsightsIAS Mock Test Series for UPSC Preliminary Exam 2020 )

52 Consider the following statements regarding Organisation for Economic Co-operation and
Development (OECD)
1. Russia is a founding member of OECD.
2. It is an intergovernmental economic organisation, founded to stimulate economic progress and
world trade.
3. Most OECD members are high-income economies with a very high Human Development Index (HDI)
and are regarded as developed countries.

Which of the statements given above is/are correct?


A. 2 only
B. 2 and 3 only
C. 3 only
D. 1 and 2 only

Correct Answer : B

Answer Justification :

Russia is not a member of OECD itself. Hence Statement 1 is incorrect.

The Organisation for Economic Co-operation and Development is an intergovernmental


economic organisation with 36 member countries, founded in 1961 to stimulate economic
progress and world trade. Hence Statement 2 is correct.

Most OECD members are high-income economies with a very high Human Development
Index (HDI) and are regarded as developed countries. Hence Statement 3 is correct.

https://www.oecd.org/about/

https://en.wikipedia.org/wiki/OECD

53 Consider the following statements regarding Raisina Dialogue


1. The Raisina Dialogue is India’s flagship annual geopolitical and geostrategic conference.
2. It is jointly organised by the Ministry of External Affairs (MEA) and Observer Research Foundation
(ORF).
3. The theme of the 2019 conference is Managing Disruptive Transitions: Ideas, Institutions and
Idioms

Which of the statements given above is/are correct?


A. 2 only
B. 1 and 3 only
C. 3 only
D. 1 and 2 only

Correct Answer : D

35
Total Marks : 200
Online Prelims TEST - 9 (SUBJECT WISE)
( InsightsIAS Mock Test Series for UPSC Preliminary Exam 2020 )

Answer Justification :

The Raisina Dialogue is India’s flagship annual geopolitical and geostrategic conference.
Statement 1 is correct.

Ministry of External Affairs in partnership with Observer Research Foundation is organizing the 4th
edition of the Raisina Dialogue in New Delhi from 08-10 January 2019. Statement 2 is correct.

2019 conference theme is "A World Reorder: New Geometries; Fluid Partnerships; Uncertain
Outcomes”. The discussions will seek to address issues arising from ongoing global transitions and
changes to the world order, triggered by unique leaders, innovative partnerships and new
technologies. Statement 3 is incorrect.

54 Member countries of Indian Ocean Rim Association (IORA) are

A. India, Egypt, Sri Lanka, Bangladesh and New Zealand


B. India, Malaysia, Saudi Arabia, South Africa and Iraq
C. India, Australia, Iran, Indonesia and Thailand
D. India, China, Singapore, Mauritius and Madagascar

Correct Answer : C

Answer Justification :

India, Australia, Iran IR, Indonesia Thailand, Malaysia, South Africa, Mozambique, Kenya, Sri
Lanka, Tanzania, Bangladesh, Singapore, Mauritius, Madagascar, UAE, Yemen, Seychelles, Somalia,
Comoros and Oman are members of IORA.

http://pib.nic.in/newsite/PrintRelease.aspx?relid=183973

55 Consider the following statements regarding International North South Transport Corridor (INSTC)
1. Multi modal transport corridor signed in 2000 in St.Petersburg.
2. Iran, Russia, India and Azerbaijan are the founding members of INSTC
3. INSTC is a part of Ashgabat agreement

Which of the statements given above is/are correct?


A. 1 and 2 only
B. 2 and 3 only
C. 1 and 3 only
D. None

Correct Answer : D

Answer Justification :

36
Total Marks : 200
Online Prelims TEST - 9 (SUBJECT WISE)
( InsightsIAS Mock Test Series for UPSC Preliminary Exam 2020 )

International North South Transport Corridor (INSTC)

• Multi modal transport corridor signed in 2000 in St. Petersburg with Iran, Russia and
India as founding members. Hence Statement 2 is incorrect.

• It was expanded to include 11 new members: Azerbaijan, Armenia, Kazakhstan, Kyrgyzstan,


Tajikistan, Turkey, Ukraine, Belarus, Oman, Syria and Bulgaria.

• It aims to link India and Iran via the sea route and then through Iran to the Caspian sea onwards
to Central Asia.

International North–South Transport Corridor will also synchronize with the Ashgabat
agreement, (not a part of it) a Multimodal transport agreement signed by India (2018), Oman
(2011), Iran (2011), Turkmenistan (2011), Uzbekistan (2011) and Kazakhstan (2015) (figure in the
bracket indicates the year of joining the agreement), for creating an international transport and
transit corridor facilitating transportation of goods between Central Asia and the Persian Gulf. This
route will be operationalized by mid-January 2018. Hence Statement 3 is incorrect.

https://en.wikipedia.org/wiki/International_North%E2%80%93South_Transport_Corridor

56 Consider the following statements

37
Total Marks : 200
Online Prelims TEST - 9 (SUBJECT WISE)
( InsightsIAS Mock Test Series for UPSC Preliminary Exam 2020 )

1. World Energy Outlook (WEO) is published by International Energy Agency


2. India is not a member country of International Energy Forum

Which of the statements given above is/are correct?


A. 1 only
B. 2 only
C. Both 1 and 2
D. Neither 1 nor 2

Correct Answer : A

Answer Justification :

The IEA flagship publication World Energy Outlook (WEO), widely regarded as the gold
standard of energy analysis. Hence Statement 1 is correct.

India hosted 16th International Energy Forum (IEF) Ministerial meet at New Delhi in April, 2018.

About International Energy Forum (IEF):

The International Energy Forum (IEF) aims to foster greater mutual understanding and
awareness of common energy interests among its members.

The 72 Member Countries of the Forum are signatories to the IEF


Charter, which outlines the framework of the global energy dialogue
through this inter-governmental arrangement.

Covering all six continents and accounting for around 90% of global supply and demand
for oil and gas, the IEF is unique in that it comprises not only consuming and producing
countries of the International Energy Agency and OPEC.

India is a member country of International Energy Forum. Hence Statement 2 is


incorrect.

The IEF is the neutral facilitator of informal, open, informed and continuing global energy
dialogue.

The Forum's biennial Ministerial Meetings are the world's largest gathering of Energy
Ministers.

The IEF and the global energy dialogue are promoted by a permanent Secretariat of
international staff based in the Diplomatic Quarter of Riyadh, Saudi Arabia.

38
Total Marks : 200
Online Prelims TEST - 9 (SUBJECT WISE)
( InsightsIAS Mock Test Series for UPSC Preliminary Exam 2020 )

https://www.ief.org/about-ief/ief-overview.aspx

https://www.ief.org/about-ief/organisation/member-countries.aspx

57 The Budapest Convention is related to which of the following matters?

A. Organic Farming
B. Ozone Depletion
C. World Trade Organization
D. Cyber Security

Correct Answer : D

Answer Justification :

The Convention on Cybercrime of the Council of Europe (CETS No.185), known as the Budapest
Convention, is the only binding international instrument on this issue.

It serves as a guideline for any country developing comprehensive national legislation against
Cybercrime and as a framework for international cooperation between State Parties to this treaty.

The Budapest Convention is supplemented by a Protocol on Xenophobia and Racism committed


through computer systems. India is not a member to the convention at present. However, recently
Ministry of Home affairs mentioned that India is reconsidering its position on becoming a member
of the Budapest Convention because of the surge in cybercrime, especially after a push for digital
India.

https://www.coe.int/en/web/cybercrime/the-budapest-convention

58 Which of the following country is not a member of Organization of the Petroleum Exporting
Countries (OPEC)?

A. Libya
B. Iran
C. Saudi Arabia
D. Qatar

Correct Answer : D

Answer Justification :

The Organization of the Petroleum Exporting Countries (OPEC) was founded in Baghdad, Iraq, with

39
Total Marks : 200
Online Prelims TEST - 9 (SUBJECT WISE)
( InsightsIAS Mock Test Series for UPSC Preliminary Exam 2020 )

the signing of an agreement in September 1960 by five countries namely Islamic Republic of
Iran, Iraq, Kuwait, Saudi Arabia and Venezuela. They were to become the Founder Members of
the Organization.
These countries were later joined by Qatar (1961), Indonesia (1962), Libya (1962), the United Arab
Emirates (1967), Algeria (1969), Nigeria (1971), Ecuador (1973), Gabon (1975), Angola (2007),
Equatorial Guinea (2017) and Congo (2018). Ecuador suspended its membership in December 1992,
but rejoined OPEC in October 2007.
Indonesia suspended its membership in January 2009, reactivated it again in January 2016, but
decided to suspend its membership once more at the 171st Meeting of the OPEC Conference on 30
November 2016. Gabon terminated its membership in January 1995. However, it rejoined the
Organization in July 2016. Qatar terminated its membership on 1 January 2019.

This means that, currently, the Organization has a total of 14 Member Countries.
https://www.opec.org/opec_web/en/about_us/25.htm

59 Which of the following organizations publishes Travel and Tourism Competitiveness Report?

A. World Economic Forum (WEF)


B. The Global work and Travels Co
C. World Travel and Tourism Council
D. Expedia Travel Agency

Correct Answer : A

Answer Justification :

The Travel and Tourism Competitiveness Report was first published in 2007 by the World
Economic Forum. The report ranks selected nations according to the Travel and Tourism
Competitiveness Index (TTCI), which scores from 1 to 6 the performance of a given country in
each specific sub-index.

https://www.weforum.org/reports/the-travel-tourism-competitiveness-report-2017

60 With reference to The National Tiger Conservation Authority, consider the following statements:
1. It is a statutory body under the Ministry of Environment, Forests and Climate Change.
2. It was constituted under provisions of the Wildlife (Protection) Act, 1972.
3. It is chaired by the Prime Minister of India.

Which of the statements given above is/are correct?


A. 1 and 2 only
B. 2 and 3 only
C. 1 and 3 only
D. 1, 2 and 3

Correct Answer : A

40
Total Marks : 200
Online Prelims TEST - 9 (SUBJECT WISE)
( InsightsIAS Mock Test Series for UPSC Preliminary Exam 2020 )

Answer Justification :

The National Tiger Conservation Authority is a statutory body under the Ministry of
Environment, Forests and Climate Change constituted under provisions of the Wildlife
(Protection) Act, 1972. Hence Statement 1 and 2 is correct.

Environment Minister is the Chairman of the NTCA. Hence Statement 3 is incorrect.

Below chairman are eight experts or professionals having qualifications and experience in
wildlife conservation and welfare of people including tribals, apart from three Members of
Parliament (1 Rajya Sabha, 2 Lok Sabha).

Objectives of NTCA are

Providing statutory authority to Project Tiger so that compliance of its directives become
legal.
Fostering accountability of Center-State in management of Tiger Reserves, by providing a
basis for MoU with States within our federal structure.
Providing for an oversight by Parliament.
Addressing livelihood interests of local people in areas surrounding Tiger Reserves.

https://projecttiger.nic.in/content/111_1_Introduction.aspx

61 Consider the following statements


1. Indus Water Treaty was brokered by World Bank in 1950.
2. As per the treaty, control over three eastern rivers Ravi, Beas and Sutlej was given to Pakistan.
3. Ratle is the run-of the-river hydroelectric power station over Jhelum River, downstream of the
village of Ratle in Jammu and Kashmir.

Which of the statements given above is/are correct?


A. 2 only
B. 3 only
C. 1 and 3 only
D. None

Correct Answer : D

Answer Justification :

All the statements given above are incorrect.

Indus Water Treaty of 1960 brokered by the World Bank. As per the treaty, control over three
eastern rivers Ravi, Beas and Sutlej was given to India.

While control over three western rivers Indus, Jhelum and Chenab was given to Pakistan.

41
Total Marks : 200
Online Prelims TEST - 9 (SUBJECT WISE)
( InsightsIAS Mock Test Series for UPSC Preliminary Exam 2020 )

Ratle (run-of-the-river hydroelectric power station) Chenab River, downstream of the village
of Ratle in Jammu and Kashmir.

62 Which of the following ideologies is closest to the “ASEAN Way”?

A. Forming formal supra-national structures and institutions


B. Settling disputes by informal cooperative methods
C. Relying on cultural rather than economic force
D. Assisting the development of Least Development Countries (LDCs)

Correct Answer : B

Answer Justification :

The ASEAN Way is defined a shared of norms, principles, and values governing the
interactions of ASEAN member states. Keeping ASEAN together despite these deep cultural,
economic and political differences - along with mistrust between different members - has always
been an immense challenge.

Unlike other regional or international organizations, ASEAN possesses its own model of diplomatic
engagement: the ASEAN Way.

63 Consider the following statements regarding Indian Ocean Conference


1. It has been initiated by Delhi based think tank India Foundation.
2. It is annual conference that aims to bring together Heads of States/Governments, Ministers,
thought leaders, scholars, diplomats, bureaucrats and practitioners from across the region on a
single platform.

Which of the statements given above is/are correct?


A. 1 only
B. 2 only
C. Both 1 and 2
D. Neither 1 nor 2

Correct Answer : C

Answer Justification :

Both the statements given above are correct.

It has been initiated by Delhi based think tank India Foundation along with its partners from
Singapore, Sri Lanka and Bangladesh.

It is annual conference that aims to bring together Heads of States/Governments,


Ministers, thought leaders, scholars, diplomats, bureaucrats and practitioners from across

42
Total Marks : 200
Online Prelims TEST - 9 (SUBJECT WISE)
( InsightsIAS Mock Test Series for UPSC Preliminary Exam 2020 )

the region on a single platform.

The 4th edition of Indian Ocean Conference (IOC) is being organised by India Foundation in
association with Government of Maldives and S. Rajaratnam School of International Studies,
Singapore on 03-04 September 2019 in Maldives. The theme of IOC 2019 is "Securing the Indian
Ocean Region: Traditional and Non-Traditional Challenges".

http://indianocean.indiafoundation.in/

64 Consider the following statements regarding Border Haats


1. They are market places organised by the two countries along the international border.
2. The border haats aim at promoting the wellbeing of the people dwelling in remote areas across the
borders of two countries.
3. Currently border haats are operational along India-Bangladesh and India-China border.

Which of the statements given above is/are correct?


A. 1 and 2 only
B. 2 only
C. 3 only
D. 2 and 3 only

Correct Answer : A

Answer Justification :

What are Border Haats?

They are market places organised by the two countries one day each week.

It is not only a market for buying daily commodities but also a reunion spot for families living on
both sides.

Aim: The border haats aim at promoting the wellbeing of the people dwelling in remote areas
across the borders of two countries, by establishing traditional system of marketing the local
produce through local markets.

The government of India and the government of Bangladesh have approved six more border haats:
two in Tripura at Palbasti and Kamalpur and four in Meghalaya at Bholaganj, Nalikata, Shibbari and
Ryngku.

India ruled out setting up trading centres along its border with China in Arunachal Pradesh saying
such an initiative could be taken only when Beijing agrees to it. Hence Statement 3 is incorrect.

https://pib.gov.in/newsite/PrintRelease.aspx?relid=155324

43
Total Marks : 200
Online Prelims TEST - 9 (SUBJECT WISE)
( InsightsIAS Mock Test Series for UPSC Preliminary Exam 2020 )

65 Consider the following statements


1. Operation Samudra Maitri is a military exercise between India and Maldives
2. Mt Everest Friendship Exercise is a military exercise between Nepal and China

Which of the statements given above is/are correct?


A. 1 only
B. 2 only
C. Both 1 and 2
D. Neither 1 nor 2

Correct Answer : B

Answer Justification :

Operation Samudra Maitri is a humanitarian assistance mission of India to help tsunami-hit


Indonesia. Three Indian Navy ships INS Tir, INS Sujatha and INS Shardul have been mobilised to
carry out Humanitarian assistance and disaster relief. Hence Statement 1 is incorrect.

The second edition of Nepal-China joint military exercise – Mt Everest Friendship Exercise-2018
also called as Sagarmatha Friendship-2018, took place in China’s southwestern Sichuan province.
Hence Statement 2 is correct.

It will focus on combating terror and disaster management training. Sagarmatha is the Nepali name
for Mt Everest, which stands in between both the countries.

66 Consider the following statements regarding Nuclear Energy Agency


1. It is an intergovernmental agency that facilitates co-operation among countries with advanced
nuclear technology infrastructures.
2. Country must ratify Non-Proliferation Treaty before joining Nuclear Energy agency.
3. It relies on member country experts to carry out much of its technical work.

Which of the statements given above is/are correct?


A. 1, 2 and 3
B. 2 and 3 only
C. 1 and 3 only
D. 1 and 2 only

Correct Answer : C

Answer Justification :

The Nuclear Energy Agency (NEA) is an intergovernmental agency that facilitates co-operation
among countries with advanced nuclear technology infrastructures to seek excellence in nuclear
safety, technology, science, environment and law. The NEA, which is under the framework of the
Organisation for Economic Co-operation and Development, is headquartered in Paris, France.

44
Total Marks : 200
Online Prelims TEST - 9 (SUBJECT WISE)
( InsightsIAS Mock Test Series for UPSC Preliminary Exam 2020 )

Hence Statement 1 is correct.

A country need not to sign or ratify Non-Proliferation Treaty before joining Nuclear Energy agency.
Hence Statement 2 is incorrect.

NEA membership represents much of the world's best nuclear expertise and it maintains
strategic partnerships with key non-member countries involved in nuclear technology.

By pooling this expertise, the NEA provides each member country access to the substantial
experience of others and an opportunity to substantially leverage its resources.

The NEA establishes a climate of mutual trust and collaboration, enabling the full exchange of
experience and a frank assessment of issues.

NEA scientific and technical work is in the forefront of knowledge and is known for its depth
and quality.

The NEA publishes consensus positions on key issues, providing member countries with
credible references.

The NEA is cost-effective, relying on member country experts to carry out much of its
technical work. Hence Statement 3 is correct.

The NEA's system of standing technical committees enables the Agency to be flexible and
responsive.

https://www.oecd-nea.org/general/about/

67 “The costs of De-carbonization” report is published by

A. United Nations Environment Programme


B. OECD
C. World Bank
D. United Nations Framework Convention on Climate Change

Correct Answer : B

Answer Justification :

Under the Paris Agreement, OECD countries agreed to aim for a reduction of their
greenhouse gas emissions sufficient to hold the increase in the global average

45
Total Marks : 200
Online Prelims TEST - 9 (SUBJECT WISE)
( InsightsIAS Mock Test Series for UPSC Preliminary Exam 2020 )

temperature to well below 2°C above pre industrial levels.

This commitment requires a massive effort to decarbonise energy and electricity generation, a
radical restructuring of the electric power sector and the rapid deployment of large amounts of low-
carbon generation technologies, in particular nuclear energy and renewable energies such as wind
and solar PV.

http://www.oecd.org/publications/the-costs-of-decarbonisation-9789264312180-en.htm

68 Which of the following country is not a member of Gulf Co-operation Council?

A. Oman
B. Qatar
C. Kuwait
D. Iran

Correct Answer : D

Answer Justification :

Gulf Cooperation Council (is a regional intergovernmental political and economic union
consisting of all Arab states of the Persian Gulf except Iraq, namely: Bahrain, Kuwait,
Oman, Qatar, Saudi Arabia, and the United Arab Emirates.

All current member states are monarchies, including three constitutional monarchies (Qatar,
Kuwait, and Bahrain), two absolute monarchies (Saudi Arabia and Oman), and one federal
monarchy (the United Arab Emirates, which is composed of seven member states, each of which is
an absolute monarchy with its own emir). There have been discussions regarding the future
membership of Jordan, Morocco, and Yemen.

46
Total Marks : 200
Online Prelims TEST - 9 (SUBJECT WISE)
( InsightsIAS Mock Test Series for UPSC Preliminary Exam 2020 )

69 Consider the following statements regarding International Energy Agency (IEA)


1. It is an autonomous intergovernmental organization established in the framework of the OECD
2. India is a member country of it.

Which of the statements given above is/are correct?


A. 1 only
B. 2 only
C. Both 1 and 2
D. Neither 1 nor 2

Correct Answer : A

47
Total Marks : 200
Online Prelims TEST - 9 (SUBJECT WISE)
( InsightsIAS Mock Test Series for UPSC Preliminary Exam 2020 )

Answer Justification :

IEA is a Paris-based autonomous intergovernmental organization established in the framework of


the Organisation for Economic Co-operation and Development (OECD) in 1974 in the wake of
the 1973 oil crisis. Hence Statement 1 is correct.

India is not a member country of International Energy Agency. Hence Statement 2 is incorrect

The IEA acts as a policy adviser to its member states, but also works with non-member
countries, especially China, India, and Russia.

The Agency's mandate has broadened to focus on the "3Es" of effectual energy policy: energy
security, economic development, and environmental protection. The latter has focused on mitigating
climate change. The IEA has a broad role in promoting alternate energy sources (including
renewable energy), rational energy policies, and multinational energy technology co-operation.

70 Consider the following statements


1. Chemical Weapons Convention (CWC) is the world’s first multilateral disarmament agreement to
provide for the elimination of an entire category of weapons of mass destruction within a fixed time
frame.
2. The Organisation for the Prohibition of Chemical Weapons (OPCW) is an intergovernmental
organisation and the implementing body for the Chemical Weapons Convention.
3. The OPCW had been awarded Nobel Peace Prize in 2013 for its extensive efforts to eliminate
chemical weapons.

Which of the statements given above is/are correct?


A. 1 and 2 only
B. 1, 2 and 3
C. 1 only
D. 2 and 3 only

Correct Answer : B

Answer Justification :

All the statements given above are correct.

History was made on 29 April 1997 with the entry into force of the Chemical Weapons Convention
(CWC)—the world’s first multilateral disarmament agreement to provide for the elimination of an
entire category of weapons of mass destruction within a fixed time frame.

The Organisation for the Prohibition of Chemical Weapons (OPCW) is the


implementing body for the Chemical Weapons Convention, which entered into force on 29 April
1997.

The OPCW, with its 193 Member States, oversees the global endeavor to permanently and verifiably
eliminate chemical weapons.

48
Total Marks : 200
Online Prelims TEST - 9 (SUBJECT WISE)
( InsightsIAS Mock Test Series for UPSC Preliminary Exam 2020 )

The 2013 Nobel Prize for Peace was awarded to the Organisation for the Prohibition of
Chemical Weapons on 10 December 2013 for ‘its extensive efforts to eliminate chemical
weapons.

71 Which of the following countries is/are members of G-20 group


1. Argentina
2. Saudi Arabia
3. Singapore
4. Spain

Select the correct answer using the code given below


A. 1 and 3 only
B. 2 and 3 only
C. 1 and 2 only
D. 1, 2 and 3 only

Correct Answer : C

Answer Justification :

The G20 comprises 19 countries and the European Union. The 19 countries are Argentina,
Australia, Brazil, Canada, China, Germany, France, India, Indonesia, Italy, Japan, Mexico, Russia,
Saudi Arabia, South Africa, South Korea, Turkey, the United Kingdom and the United States

49
Total Marks : 200
Online Prelims TEST - 9 (SUBJECT WISE)
( InsightsIAS Mock Test Series for UPSC Preliminary Exam 2020 )

https://g20.org/en/summit/about/

72 Consider the following statements regarding International Solar Alliance


1. The initiative was launched at the UN Climate Change Conference in Paris at the end of 2015 by the
President of France and the Prime Minister of India.
2. The primary aim of this is to provide quick and easy access to solar resource data globally.
3. Countries that do not fall within the Tropics cannot join the alliance.

Which of the statements given above is/are correct?


A. 2 and 3 only
B. 1 and 2 only
C. 1, 2 and 3
D. None

Correct Answer : B

Answer Justification :

50
Total Marks : 200
Online Prelims TEST - 9 (SUBJECT WISE)
( InsightsIAS Mock Test Series for UPSC Preliminary Exam 2020 )

The initiative was launched at the UN Climate Change Conference in Paris at the end of 2015 by the
President of France and the Prime Minister of India. Hence Statement 1 is correct.

The primary aim of this is to provide quick and easy access to solar resource data globally. Hence
Statement 2 is correct.

The alliance is a treaty-based inter-governmental organization. Countries that do not fall


within the Tropics can join the alliance and enjoy all benefits as other members, with the
exception of voting rights. Hence Statement 3 is incorrect.

https://en.wikipedia.org/wiki/International_Solar_Alliance

73 Which of the following country is not an observer of SAARC?


#00000

A. Australia
B. France
C. European Union
D. China

Correct Answer : B

Answer Justification :

States with observer status include Australia, China, the European Union, Iran, Japan,
Mauritius, Myanmar, South Korea and the United States.

On 2 August 2006, the foreign ministers of the SAARC countries agreed in principle to grant
observer status to three applicants; the US and South Korea (both made requests in April 2006), as
well as the European Union (requested in July 2006). On 4 March 2007, Iran requested observer
status, followed shortly by Mauritius.

https://en.wikipedia.org/wiki/South_Asian_Association_for_Regional_Cooperation#Observers

74 Consider the following statements regarding Treaty on the Prohibition of Nuclear Weapons
1. It includes a comprehensive set of prohibitions on participating in any nuclear weapon activities.
2. The Treaty obliges States parties to provide adequate assistance to individuals affected by the use
or testing of nuclear weapons.
3. It will enter into force 90 days after the fiftieth instrument of ratification, acceptance, approval or
accession has been deposited.

Which of the statements given above is/are correct?


A. 2 and 3 only
B. 1 and 2 only
C. 1, 2 and 3
D. None

51
Total Marks : 200
Online Prelims TEST - 9 (SUBJECT WISE)
( InsightsIAS Mock Test Series for UPSC Preliminary Exam 2020 )

Correct Answer : C

Answer Justification :

All the statements given above are correct.

The Treaty on the Prohibition of Nuclear Weapons (TPNW) includes a comprehensive set
of prohibitions on participating in any nuclear weapon activities. These include undertakings
not to develop, test, produce, acquire, possess, stockpile, use or threaten to use nuclear weapons.

The Treaty also prohibits the deployment of nuclear weapons on national territory and the provision
of assistance to any State in the conduct of prohibited activities. States parties will be obliged to
prevent and suppress any activity prohibited under the TPNW undertaken by persons or on territory
under its jurisdiction or control.

The Treaty also obliges States parties to provide adequate assistance to individuals
affected by the use or testing of nuclear weapons, as well as to take necessary and appropriate
measure of environmental remediation in areas under its jurisdiction or control contaminated as a
result of activities related to the testing or use of nuclear weapons.

The Treaty on the Prohibition of Nuclear Weapons was adopted by the Conference (by a vote of 122
States in favour (with one vote against and one abstention) at the United Nations on 7 July 2017,
and opened for signature by the Secretary-General of the United Nations on 20 September 2017.

It will enter into force 90 days after the fiftieth instrument of ratification, acceptance,
approval or accession has been deposited.

https://www.un.org/disarmament/wmd/nuclear/tpnw/

75 Consider the following statements


1. The European Parliament is the only legislative branch of the European Union.
2. Council of the European Union is the executive branch of the European Union, responsible for
proposing legislation, implementing decisions, upholding the EU treaties and managing the day-to-
day business of the EU.

Which of the statements given above is/are correct?


A. 1 only
B. 2 only
C. Both 1 and 2
D. Neither 1 nor 2

Correct Answer : D

Answer Justification :

Both the statements are incorrect.

52
Total Marks : 200
Online Prelims TEST - 9 (SUBJECT WISE)
( InsightsIAS Mock Test Series for UPSC Preliminary Exam 2020 )

The European Parliament (EP) is the legislative branch of the European Union and one of
its seven institutions. Together with the Council of the European Union, it adopts
European legislation, normally on a proposal from the European Commission.

The Parliament is composed of 751 members (MEPs), intended to become 705 starting from the
2019–2024 legislature because of specific provisions adopted about Brexit, who represent the
second-largest democratic electorate in the world (after the Parliament of India) and the largest
trans-national democratic electorate in the world (375 million eligible voters in 2009).

The European Commission (EC) is the executive branch of the European Union,
responsible for proposing legislation, implementing decisions, upholding the EU treaties
and managing the day-to-day business of the EU.

https://en.wikipedia.org/wiki/European_Parliament

https://en.wikipedia.org/wiki/European_Commission

76 Consider the following pairs


Place/Issue in News Region
1. Port of Duqm Yemen
2. Gilets Jaunes Protests France
3. Farzad-B Gas Block Iran

Which of the pairs given above is/are correctly matched?


A. 3 only
B. 1 and 3 only
C. 1 and 2 only
D. 2 and 3 only

Correct Answer : D

Answer Justification :

Port of Duqm is located in Oman.

The yellow vests movement or yellow jackets movement is a populist, grassroots


revolutionary political movement for economic justice that began in France in October
2018. After an online petition posted in May had attracted nearly a million signatures, mass
demonstrations began on 17 November.

The Farzad B gas field is an Iranian natural gas field that was discovered in 2012. It began
production in 2013 and produces natural gas and condensates.

53
Total Marks : 200
Online Prelims TEST - 9 (SUBJECT WISE)
( InsightsIAS Mock Test Series for UPSC Preliminary Exam 2020 )

77 Consider the following pairs


Issue in News Region
1. Changi Naval Base Indonesia
2. Tripoli Libya
3. Male Maldives

Which of the statements given above is/are correct?


A. 2 only
B. 2 and 3 only
C. 1 and 3 only
D. 3 only

Correct Answer : B

Answer Justification :

Changi Naval Base is located in Singapore.

Tripoli is capital of Libya. Tripoli is a city located in Lebanon also.

Malé is the capital and most populous city in the Republic of Maldives.

78 Consider the following pairs


Militia sometimes in the news Country they belong to
1. People’s Protection Units Turkey
2. Revolutionary Guard Iraq
3. Salwa Judum India

Which of the pairs given above is/are correctly matched?


A. 1 only
B. 1 and 3 only
C. 2 and 3 only
D. 3 only

Correct Answer : D

Answer Justification :

The YPG or People’s Protection Units are the armed wing of the Syrian Kurdish
Democratic Union Party.

The Islamic Revolutionary Guard Corps is a branch of Iran’s Armed Forces founded after
the 1979 Revolution by order of Ayatollah Khomeini.

54
Total Marks : 200
Online Prelims TEST - 9 (SUBJECT WISE)
( InsightsIAS Mock Test Series for UPSC Preliminary Exam 2020 )

Salwa Judum was a militia that was mobilised and deployed as part of anti-insurgency
operations in Chhattisgarh, India, aimed at countering Naxalite violence in the region.

79 Consider the following statements regarding Quadrilateral Security Dialogue Summit


1. It is an informal mechanism between India, the US, Japan and South Korea
2. It is interpreted as a joint effort to counter China’s influence in the Indo-Pacific region.

Which of the statements given above is/are correct?


A. 1 only
B. 2 only
C. Both 1 and 2
D. Neither 1 nor 2

Correct Answer : B

Answer Justification :

It is an informal mechanism between India, the US, Australia and Japan. Hence Statement
1 is incorrect

It is interpreted as a joint effort to counter China’s influence in the Indo-Pacific region.


Hence Statement 2 is correct.

The idea of the Quad could be originally attributed to Japanese Prime Minister Shinzo Abe.

https://foreignpolicy.com/2018/07/23/india-is-the-weakest-link-in-the-quad/

80 Consider the following statements regarding United Nations Human Rights Council
1. It has 47 members elected for staggered three-year terms on a regional group basis.
2. It was established by the UN General Assembly to replace the UN Commission on Human Rights.

Which of the statements given above is/are correct?


A. 1 only
B. 2 only
C. Both 1 and 2
D. Neither 1 nor 2

Correct Answer : C

Answer Justification :

Both the statements are correct.

The United Nations Human Rights Council is a United Nations body whose mission is to
promote and protect human rights around the world.

55
Total Marks : 200
Online Prelims TEST - 9 (SUBJECT WISE)
( InsightsIAS Mock Test Series for UPSC Preliminary Exam 2020 )

The UNHRC has 47 members elected for staggered three-year terms on a regional group
basis. The headquarters of UNHRC is in Geneva, Switzerland.

The UNHRC investigates allegations of breaches of human rights in UN member states, and
addresses important thematic human rights issues such as freedom of association and assembly,
freedom of expression, freedom of belief and religion, women's rights, LGBT rights, and the rights
of racial and ethnic minorities.

The UNHRC was established by the UN General Assembly on March 15, 2006 (by
resolution A/RES/60/251) to replace the UN Commission on Human Rights (UNCHR,
herein CHR) that had been strongly criticised for allowing countries with poor human rights
records to be members

https://en.wikipedia.org/wiki/United_Nations_Human_Rights_Council

81 Consider the following statements regarding TRAFFIC


1. It is a joint program of WWF and UNEP
2. TRAFFIC works to ensure that trade in wild plants and animals is not a threat to the conservation of
nature.

Which of the statements given above is/are correct?


A. 1 only
B. 2 only
C. Both 1 and 2
D. Neither 1 nor 2

Correct Answer : B

Answer Justification :

TRAFFIC, the wildlife trade monitoring network, is a joint program of WWF and IUCN –
the International Union for Conservation of Nature. Hence Statement 1 is incorrect

TRAFFIC works to ensure that trade in wild plants and animals is not a threat to the
conservation of nature. TRAFFIC has gained its greatest reputation from supporting CITES, the
Convention on International Trade in Endangered Species. Since TRAFFIC was created in 1976 it
has helped with the evolution of this international wildlife trade treaty. TRAFFIC North America
(NA) works through its offices in the United States, Canada and Mexico, and with its programs in
Central America and the Caribbean. Hence Statement 2 is correct.

TRAFFIC focuses on leveraging resources, expertise and awareness of the latest globally urgent
species trade issues such as tiger parts, elephant ivory and rhino horn. Large scale commercial
trade in commodities like timber and fisheries products are also addressed and linked to work on
developing rapid results and policy improvements.

https://www.worldwildlife.org/initiatives/traffic-the-wildlife-trade-monitoring-network

56
Total Marks : 200
Online Prelims TEST - 9 (SUBJECT WISE)
( InsightsIAS Mock Test Series for UPSC Preliminary Exam 2020 )

82 Consider the following statements regarding National Policy on Bio-fuels


1. The Policy encourages setting up of supply chain mechanisms for biodiesel production from non-
edible oilseeds.
2. Policy prohibits the use of surplus food grains for production of ethanol for blending with petrol
citing food security.
3. Policy indicates a viability gap funding scheme ethanol Bio refineries.

Which of the statements given above is/are correct?


A. 1 and 3 only
B. 2 and 3 only
C. 1 only
D. 1, 2 and 3

Correct Answer : A

Answer Justification :

Salient Features of National Policy on Biofuels – 2018

1. The Policy categorises biofuels as "Basic Biofuels" viz. First Generation (1G) bioethanol &
biodiesel and "Advanced Biofuels" - Second Generation (2G) ethanol, Municipal Solid Waste
(MSW) to drop-in fuels, Third Generation (3G) biofuels, bio-CNG etc. to enable extension of
appropriate financial and fiscal incentives under each category.

2. The Policy expands the scope of raw material for ethanol production by allowing use of
Sugarcane Juice, Sugar containing materials like Sugar Beet, Sweet Sorghum, Starch
containing materials like Corn, Cassava, Damaged food grains like wheat, broken rice, Rotten
Potatoes, unfit for human consumption for ethanol production.

3. Farmers are at a risk of not getting appropriate price for their produce during the surplus
production phase. Taking this into account, the Policy allows use of surplus food
grains for production of ethanol for blending with petrol with the approval of
National Biofuel Coordination Committee. Hence Statement 2 is incorrect.

4. With a thrust on Advanced Biofuels, the Policy indicates a viability gap funding scheme for 2G
ethanol Bio refineries of Rs.5000 crore in 6 years in addition to additional tax incentives,
higher purchase price as compared to 1G biofuels.

5. The Policy encourages setting up of supply chain mechanisms for biodiesel production from
non-edible oilseeds, Used Cooking Oil, short gestation crops.

6. Roles and responsibilities of all the concerned Ministries/Departments with respect to biofuels
has been captured in the Policy document to synergise efforts.

57
Total Marks : 200
Online Prelims TEST - 9 (SUBJECT WISE)
( InsightsIAS Mock Test Series for UPSC Preliminary Exam 2020 )

https://pib.gov.in/Pressreleaseshare.aspx?PRID=1532265

83 Consider the following statements


1. Krishna River originates at Mahabaleshwar in Maharashtra.
2. Krishna Water Dispute Tribunal was first setup in 1969.
3. Krishna River joins Bay of Bengal near Chennai

Which of the statements given above is/are correct?


A. 1, 2 and 3
B. 2 and 3 only
C. 1 only
D. 1 and 2 only

Correct Answer : D

Answer Justification :

The Krishna is an east-flowing river that originates at Mahabaleshwar in Maharashtra and merges
with the Bay of Bengal, flowing through Maharashtra, Karnataka, Telangana and Andhra Pradesh.
Together with its tributaries, it forms a vast basin that covers 33% of the total area of the four
states. Hence Statement 1 is correct.

In 1969, the Krishna Water Disputes Tribunal (KWDT) was set up under the Inter-State River Water
Dispute Act, 1956, and presented its report in 1973. The report, which was published in 1976,
divided the 2060 TMC (thousand million cubic feet) of Krishna water at 75 per cent dependability
into three parts: 560 TMC for Maharashtra, 700 TMC for Karnataka and 800 TMC for Andhra
Pradesh. Hence Statement 2 is correct.

Krishna River empties into the Bay of Bengal at Hamasaladeevi (near Koduru) in Andhra
Pradesh. Hence Statement 3 is incorrect.

https://indianexpress.com/article/explained/explained-what-is-the-krishna-water-dispute-and-who-all
-are-involved-5975736/

84 Consider the following statements regarding United Nations Convention to Combat Desertification
(UNCCD)
1. UNCCD was adopted in 1994 and ratified by nearly 100 countries
2. It is committed to a bottom-up approach, encouraging the participation of local people in combating
desertification and land degradation.
3. India takes over COP Presidency from China for next two years

Which of the statements given above is/are correct?


A. 1 and 2 only
B. 2 only
C. 1 only
D. 2 and 3 only

58
Total Marks : 200
Online Prelims TEST - 9 (SUBJECT WISE)
( InsightsIAS Mock Test Series for UPSC Preliminary Exam 2020 )

Correct Answer : D

Answer Justification :

UNCCD was adopted in Paris on 17 June 1994 and ratified by 196 countries & European
Union. India ratified the UNCCD Convention on December 1996. Hence Statement 1 is
incorrect.

It is committed to a bottom-up approach, encouraging the participation of local people in combating


desertification and land degradation. Hence Statement 2 is correct.

India takes over COP Presidency of UNCCD from China for next two years. Through hosting COP
14, India will highlight its leadership in navigating the land management agenda at global level. It
will also provide a stage to mainstream sustainable land management in country’s national
development policies. Hence Statement 3 is correct.

https://www.pib.nic.in/PressReleasePage.aspx?PRID=1583898

85 Consider the following statements regarding Bombay blood group


1. Individuals with the rare Bombay phenotype (hh) do not express H antigen.
2. It is mostly found in South Asia and South East Asia.

Which of the statements given above is/are correct?


A. 1 only
B. 2 only
C. Both 1 and 2
D. Neither 1 nor 2

Correct Answer : A

Answer Justification :

The h/h blood group, also known as Oh or the Bombay blood group, is a rare blood type. This blood
phenotype was first discovered in Bombay, now known as Mumbai, in India, by Dr. Y. M. Bhende in
1952. It is mostly found in South Asia (India, Bangladesh, Pakistan) and parts of Middle East such
as Iran. Hence Statement 1 is correct.

Individuals with the rare Bombay phenotype (hh) do not express H antigen (also called substance
H), the antigen which is present in blood group O. As a result, they cannot make A antigen (also
called substance A) or B antigen (substance B) on their red blood cells, whatever alleles they may
have of the A and B blood-group genes, because A antigen and B antigen are made from H antigen.
Hence Statement 2 is incorrect.

https://en.wikipedia.org/wiki/Hh_blood_group

https://indianexpress.com/article/explained/what-is-bombay-blood-group-rare-and-sought-after-5984

59
Total Marks : 200
Online Prelims TEST - 9 (SUBJECT WISE)
( InsightsIAS Mock Test Series for UPSC Preliminary Exam 2020 )

653/

86 Which of the following tribes is/are present in Ladakh Region


1. Balti
2. Changpa
3. Purigpa

Select the correct answer using the code given below


A. 1 and 2 only
B. 2 only
C. 2 and 3 only
D. 1, 2 and 3

Correct Answer : D

Answer Justification :

The total tribal population in Ladakh region is more than 97%. The region is inhabited by following
Scheduled Tribes, namely:

Balti
Beda
Bot, Boto
Brokpa, Drokpa, Dard, Shin
Changpa
Garra
Mon
Purigpa

https://www.insightsonindia.com/2019/09/12/insights-daily-current-affairs-pib-12-september-2019/

87 Consider the following statements regarding Government e Marketplace (GeM)


1. It is a one stop portal to facilitate online procurement of common use Goods & Services required by
various Government Departments.
2. Only Central government Ministries/Departments including its attached/subordinate offices are
eligible to procure goods from its platform

Which of the statements given above is/are correct?


A. 1 only
B. 2 only
C. Both 1 and 2
D. Neither 1 nor 2

Correct Answer : A

60
Total Marks : 200
Online Prelims TEST - 9 (SUBJECT WISE)
( InsightsIAS Mock Test Series for UPSC Preliminary Exam 2020 )

Answer Justification :

Government e-Marketplace (GeM) is a one stop portal to facilitate online procurement of common
use Goods & Services required by various Government Departments / Organizations / PSUs. GeM
aims to enhance transparency, efficiency and speed in public procurement.

It provides the tools of e-bidding, reverse e-auction and demand aggregation to facilitate the
government users achieve the best value for their money. Hence Statement 1 is correct.

Who can buy/purchase through GeM?

All Central government and State Government Ministries/Departments including its


attached/subordinate offices, Central and State autonomous bodies, Central and State Public Sector
Units and local bodies etc. are authorized to make procurement through GeM portal. Hence
Statement 2 is incorrect.

http://vikaspedia.in/e-governance/online-citizen-services/government-to-business-services-g2b/gover
nment-e-marketplace

88 Consider the following statements regarding Eurasian economic union


1. It is an international organization for regional economic integration.
2. Russia, Belarus, Armenia and China are members of it.

Which of the statements given above is/are correct?


A. 1 only
B. 2 only
C. Both 1 and 2
D. Neither 1 nor 2

Correct Answer : A

Answer Justification :

About Eurasian economic union:

It is an international organization for regional economic integration.


It has international legal personality and is established by the Treaty on the Eurasian
Economic Union.
Composition: Includes Russia, Belarus, Armenia, Kyrgyzstan and Kazakhstan. China
is not a member.
It has free movement of goods, services and labour.
It has its own bureaucratic structure.

61
Total Marks : 200
Online Prelims TEST - 9 (SUBJECT WISE)
( InsightsIAS Mock Test Series for UPSC Preliminary Exam 2020 )

https://www.insightsonindia.com/2019/09/12/insights-daily-current-affairs-pib-12-september-2019/

89 Consider the following statements regarding ASEAN.


1. ASEAN was preceded by an organization formed in 31 July 1961 called the Association of Southeast
Asia (ASA), a group consisting of the Philippines, the Federation of Malaya, and Thailand.
2. Vietnam is not a member of ASEAN.
3. ASEAN established Asian Development Bank for the socio-economic development of Asia and
Pacific.

Which of the statements given above is/are correct?


A. 1 and 3 only
B. 2 and 3 only
C. 1 only
D. None

Correct Answer : C

Answer Justification :

The Association of Southeast Asian Nations is a regional intergovernmental organization comprising


ten countries in Southeast Asia, which promotes intergovernmental cooperation and facilitates

62
Total Marks : 200
Online Prelims TEST - 9 (SUBJECT WISE)
( InsightsIAS Mock Test Series for UPSC Preliminary Exam 2020 )

economic, political, security, military, educational, and sociocultural integration among its members
and other countries in Asia. Hence Statement 1 is correct.

ASEAN was preceded by an organisation formed on 31 July 1961 called the Association of Southeast
Asia (ASA), a group consisting of Thailand, the Philippines, and the Federation of Malaya. ASEAN
itself was created on 8 August 1967, when the foreign ministers of five countries: Indonesia,
Malaysia, the Philippines, Singapore, and Thailand, signed the ASEAN Declaration.

Vietnam is a member of ASEAN. Hence Statement 2 is incorrect.

The Asian Development Bank (ADB) is a regional development bank established on 19 December
1966. It is not established by ASEAN. Hence Statement 3 is incorrect.

https://asean.org/asean/about-asean/

90 Consider the following statements regarding International Tribunal for the Law of the Sea (ITLOS)
1. ITLOS is an independent judicial body established by the UNCLOS to adjudicate disputes arising
out of the interpretation and application of the UNCLOS Convention.
2. The Tribunal is open only to the states parties to the convention.

Which of the statements given above is/are correct?


A. 1 only
B. 2 only
C. Both 1 and 2
D. Neither 1 nor 2

Correct Answer : A

Answer Justification :

International Tribunal for the Law of the Sea is an independent judicial body established by the
UNCLOS to adjudicate disputes arising out of the interpretation and application of the UNCLOS
Convention. Hence Statement 1 is correct.

HQ - Hamburg, Germany.

The Tribunal is open to States Parties to the Convention.

It is also open to entities other than States Parties, i.e., States or intergovernmental
organisations which are not parties to the Convention and to state enterprises and private entities.
Hence Statement 2 is incorrect.

Recently, Dr. Neeru Chadha has become the first Indian woman to be elected as a judge at the
International Tribunal for the Law of the Seas (ITLOS).

https://www.itlos.org/the-tribunal/

63
Total Marks : 200
Online Prelims TEST - 9 (SUBJECT WISE)
( InsightsIAS Mock Test Series for UPSC Preliminary Exam 2020 )

91 Consider the following statements regarding Fall Armyworm


1. It is an invasive species that is native to tropical and subtropical regions of the Americas.
2. It significantly impacted the productivity of coffee plantation in Western Ghats region.

Which of the statements given above is/are correct?


A. 1 only
B. 2 only
C. Both 1 and 2
D. Neither 1 nor 2

Correct Answer : A

Answer Justification :

What is Fall Armyworm (FAW)?

It is a native of the tropical and sub-tropical regions of the Americas. Hence Statement 1 is
correct.

First detected in the African continent in 2016. Since then, it has spread to other countries such as
China, Thailand, Malaysia and Sri Lanka.

In the absence of natural control or good management, it can cause significant damage to crops. It
prefers maize, but can feed on more than 80 additional species of crops, including rice, sorghum,
millet, sugarcane, vegetable crops and cotton. There is no substantial evidence of its impact on
Coffee Plantation in Western Ghats Region. Hence Statement 2 is incorrect.

92 Which of the following countries have signed The Joint Comprehensive Plan of Action with Iran in
2015?
1. U.S.A
2. U.K
3. Germany
4. China

Select the correct answer using the code given below


A. 1 and 4 only
B. 1 and 2 only
C. 1. 2, 3 and 4
D. 2, 3 and 4 only

Correct Answer : C

Answer Justification :

On July 14, 2015, the P5+1 (China, France, Germany, Russia, the United Kingdom, and the
United States), the European Union (EU), and Iran reached a Joint Comprehensive Plan of

64
Total Marks : 200
Online Prelims TEST - 9 (SUBJECT WISE)
( InsightsIAS Mock Test Series for UPSC Preliminary Exam 2020 )

Action (JCPOA) to ensure that Iran’s nuclear program will be exclusively peaceful.

https://www.armscontrol.org/factsheets/JCPOA-at-a-glance

93 Consider the following statements regarding Indian ports


1. Marmagao Port, situated at the entrance of the Zuari estuary, is a natural harbour in Goa.
2. Visakhapatnam Port in Andhra Pradesh is a land-locked harbour
3. The Kamarajar Port Limited is a corporatized major port and is registered as a public company.

Which of the statements given above is/are correct?


A. 1 only
B. 2 and 3 only
C. 1 and 2 only
D. 1, 2 and 3

Correct Answer : D

Answer Justification :

All the statements given above are correct.

Marmagao Port, situated at the entrance of the Zuari estuary, is a natural harbour in Goa.
It gained significance after its remodelling in 1961 to handle iron-ore exports to Japan. Construction
of Konkan railway has considerably extended the hinterland of this port. Karnataka, Goa, Southern
Maharashtra constitute its hinterland.

Visakhapatnam Port in Andhra Pradesh is a land-locked harbour, connected to the sea by a


channel cut through solid rock and sand. An outer harbour has been developed for handling iron-
ore, petroleum and general cargo. Andhra Pradesh and Telangana are the main hinterland for this
port.

Ennore Port, officially renamed Kamarajar Port Limited, is located on the Coromandel
Coast about 24 km north of Chennai Port, Chennai, it is the 12th major port of India, and the first
port in India which is a public company.

94 Which of the following Indian cities are a part of UNESCO's Creative Cities Network?
1. Jaipur
2. Mysore
3. Varanasi
4. Chennai

Select the correct answer using the code given below.


A. 1 and 3 only
B. 1, 3 and 4 only
C. 2, 3 and 4 only
D. 1, 2, 3 and 4

65
Total Marks : 200
Online Prelims TEST - 9 (SUBJECT WISE)
( InsightsIAS Mock Test Series for UPSC Preliminary Exam 2020 )

Correct Answer : B

Answer Justification :

The UNESCO Creative Cities Network (UCCN) was created in 2004 to promote cooperation with
and among cities that have identified creativity as a strategic factor for sustainable urban
development.

The Network covers seven creative fields: Crafts and Folk Arts, Media Arts, Film, Design,
Gastronomy, Literature and Music.

The Creative Cities Network is a privileged partner of UNESCO, not only as a platform for reflection
on the role of creativity as a lever for sustainable development but also as a breeding ground of
action and innovation, notably for the implementation of the 2030 Agenda for Sustainable
Development.

Two Indian cities for first time have been designated as members of UNESCO's Creative Cities
Network (UCCN). 1. Varanasi City (Uttar Pradesh): It has been added in the City of Music
category of network and 2. Jaipur City (Rajasthan): It has been added in the City of Crafts and
Folk Art category of network.

Chennai has been included in the UNESCO Creative Cities Network for its rich musical
tradition. Including Chennai, a total of 64 cities from 44 countries have joined the UNESCO
Creative Cities Network.

https://en.unesco.org/about-us/introducing-unesco

95 The ‘Women Entrepreneurship Platform’, a first of its kind, unified access portal which brings
together women from different parts of India to realize their entrepreneurial aspirations, is an
initiative of

A. SIDBI (Small Industries Development Bank of India)


B. Ministry of Women and Child Development
C. FICCI (Federation of Indian Chambers of Commerce and Industry)
D. NITI Aayog

Correct Answer : D

Answer Justification :

The Women Entrepreneurship Platform (WEP)

NITI Aayog has launched a Women Entrepreneurship Platform (WEP) for providing an ecosystem
for budding & existing women entrepreneurs across the country. SIDBI has partnered with NITI
Aayog to assist in this initiative.

As an enabling platform, WEP is built on three pillars- Iccha Shakti, Gyaan Shakti & Karma Shakti

66
Total Marks : 200
Online Prelims TEST - 9 (SUBJECT WISE)
( InsightsIAS Mock Test Series for UPSC Preliminary Exam 2020 )

Iccha Shakti represents motivating aspiring entrepreneurs to start their business.


Gyaan Shakti represents providing knowledge and ecosystem support to women
entrepreneurs to help them foster entrepreneurship.
Karma Shakti represents providing hands-on support to entrepreneurs in setting-up and
scaling up businesses.

https://www.startupindia.gov.in/content/sih/en/government-schemes/Wep.html

96 Consider the following statements


1. Sangai is the state animal of Mizoram and largely found in Keibul Lamjao National Park
2. Leptospirosis is a viral infection in rodents and other wild and domesticated species, and humans.

Which of the statements given above is/are correct?


A. 1 only
B. 2 only
C. Both 1 and 2
D. Neither 1 nor 2

Correct Answer : D

Answer Justification :

Both the statements are incorrect.

Sangai is the state animal of Manipur and largely found in Keibul Lamjao National Park.

Leptospirosis is a bacterial infection in rodents and other wild and domesticated species, and
humans.

97 Which of the following is/are condition(s) for recognizing a party as National Party?
1. If it wins two per cent of seats in the Lok Sabha at a general election; and these candidates are
elected from three states.
2. If it is recognized as a state party in four states.
3. If it wins three per cent of seats in the legislative assembly at a general election to the legislative
assembly of the state concerned or 3 seats in the assembly, whichever is more.

Which of the statements given above is/are correct?


A. 1 and 2 only
B. 2 only
C. 3 only
D. 1, 2 and 3

Correct Answer : A

Answer Justification :

67
Total Marks : 200
Online Prelims TEST - 9 (SUBJECT WISE)
( InsightsIAS Mock Test Series for UPSC Preliminary Exam 2020 )

Party is recognised as a national party if any of the following conditions is fulfilled:

1. If it secures six per cent of valid votes polled in any four or more states at a general election to
the Lok Sabha or to the legislative assembly; and, in addition, it wins four seats in the Lok Sabha
from any state or states; or

2. If it wins two per cent of seats in the Lok Sabha at a general election; and these candidates are
elected from three states; or

3. If it is recognised as a state party in four states

A party is recognised as a state party in a state if any of the following conditions is


fulfilled:

1. If it secures six per cent of the valid votes polled in the state at a general election to the
legislative assembly of the state concerned; and, in addition, it wins 2 seats in the assembly of the
state concerned; or

2. If it secures six per cent of the valid votes polled in the state at a general election to the Lok
Sabha from the state concerned; and, in addition, it wins 1 seat in the Lok Sabha from the state
concerned; or

3. If it wins three per cent of seats in the legislative assembly at a general election to the legislative
assembly of the state concerned or 3 seats in the assembly, whichever is more; or

4. If it wins 1 seat in the Lok Sabha for every 25 seats or any fraction thereof allotted to the state at
a general election to the Lok Sabha from the state concerned; or

5. If it secures eight per cent of the total valid votes polled in the state at a General Election to the
Lok Sabha from the state or to the legislative assembly of the state. This condition was added in
2011.

98 Consider the following statements


1. Project CHAMAN is a pioneer project aims for the strategic development of the fisheries sector.
2. Mission Fingerling is a programme aimed at achieving holistic development of horticulture sector.

Which of the statements given above is/are correct?


A. 1 only
B. 2 only
C. Both 1 and 2
D. Neither 1 and 2

Correct Answer : D

68
Total Marks : 200
Online Prelims TEST - 9 (SUBJECT WISE)
( InsightsIAS Mock Test Series for UPSC Preliminary Exam 2020 )

Answer Justification :

Both the statements are incorrect.

Project CHAMAN is a pioneer project to provide strategic development to the horticulture


sector, so as to increase farmer’s income. This project is being implemented by National Crop
Forecast Centre (MNCFC) using remote sensing technology and is likely to be completed in March
2018 in all the states.

Mission Fingerling, a programme aimed at achieving the Blue Revolution by enabling


holistic development and management of fisheries.

99 Okjokull glacier, recently seen in news is located in

A. Canada
B. Greenland
C. Antarctica
D. Iceland

Correct Answer : D

Answer Justification :

The Okjokull glacier became the first Icelandic glacier lost due to climate change.

https://indianexpress.com/article/trending/trending-globally/memorial-plaque-installed-at-okjokull-gl
acier-the-first-one-in-iceland-lost-to-climate-change-5917394/

100 Consider the following statements


1. Port Blair is closer to Kolkata than it is to Chennai.
2. Duncan passage separates Middle Andaman from South Andaman.

Which of the statements given above is/are correct?


A. 1 only
B. 2 only
C. Both 1 and 2
D. Neither 1 nor 2

Correct Answer : A

Answer Justification :

69
Total Marks : 200
Online Prelims TEST - 9 (SUBJECT WISE)
( InsightsIAS Mock Test Series for UPSC Preliminary Exam 2020 )

Duncan pass is located between South and little Andaman. It is a strait in the Indian Ocean. It is
about 48 km wide; it separates Rutland Island to the north, and Little Andaman to the south. West
of Duncan Passage is the Bay of Bengal; east is the Andaman Sea. Several small islands and islets
lie along the passage.

70

You might also like